Você está na página 1de 88

2009FootandAnkleSelfAssessmentExamination

EvaluationandCME

ACCREDITATION
TheAmericanAcademyofOrthopaedicSurgeonsisaccreditedbytheAccreditationCouncilforContinuingMedical
Educationtoprovidecontinuingmedicaleducationforphysicians.

CMECREDIT
U.S.Physicians:TheAmericanAcademyofOrthopaedicSurgeonsdesignatesthiseducationalactivityforamaximum
of10AMAPRACategory1Credits.Physiciansshouldonlyclaimcreditscommensuratewiththeextentoftheir
participationintheactivity.

InternationalPhysicians:TheAMAhasdeterminedthatphysiciansnotlicensedintheUnitedStatesbutwho
participateinthisCMEactivityareeligibleforAMAPRACategory1Credits.

AlliedHealthProfessionals:TheAcademyisnotaccreditedtooffercreditfornursesandotherallied
healthprofessionals.TodetermineifactivitiesofferingCategory1CMEcreditareacceptableforyour
licensingorcertificationneedspleasecontacttherelevantorganizationsdirectly.

ToobtainAMAPRACategory1CreditsTMforthisexamination,pleasecompletetheonlineevaluationformandsubmit
nolaterthanDecember31,2011.

1. Thiseducationalprogram(checkallthatapply)

confirmedthatmyknowledgeinthissubjectareaisuptodate
presentedmewithnewknowledgeonthetopic
presentedmewithnewknowledgedirectlyapplicabletomypractice
stimulatedmetogainfurtherinformationonthetopic

2. Didthe2009FootandAnkleSelfAssessmentExaminationmeetthestatedobjectives?

Differentiateamongtreatmentoptionsformanagingpatientswithpaininthefootandankle
Yes NoExplain:

Differentiateamongtreatmentoptionsforpatientswithdeformitiesofthefoot
Yes NoExplain:

Applyknowledgeoffootandankleanatomyindeterminingsurgicalexposuresfortheforefoot,midfoot,
hindfoot,andankle
Yes NoExplain:

Determinemethodstotreatmusculoskeletaltraumaofthefootandankle
Yes NoExplain:

Developtreatmentplansforpatientswithdiabetesmellitus,tendonandligamentruptures,andneuropathiesof
thefootandankle
Yes NoExplain:
Determinemethodstotreatsofttissueinjuriesinthefootandankle
Yes NoExplain:

3. Didyoufindtheeducationalcontentofthisexaminationmetyourlearningneedsinthiscontentarea?

Fellshortofmyexpectations
Metmyexpectations
Exceededmyexpectations

4. Towhatextentwillyouimplementknowledgegainedthroughthe2009FootandAnkleSelfAssessment
Examinationactivityintoyourpracticewithinthenextyear?

Notatall
Someextent
Moderateextent
Greatextent

Listthechangesyouwillmakeinyourpracticewithinthenextyearasaresultofcompletingthisexaminationprogram:

5. Wastheeducationalvalueofthisexaminationcompromisedbyauthorbias?

Yes
No

Explain:

6. Didyouobserveanyinstance(s)ofcommercialbiasintheexamination(i.e.,useofbrandnamesvs.generic;
favoritismtowardonecompanysdevicesvs.allothers)?Ifso,pleaseindicatethequestion(s)below. Yes
No

Explain:

Pleaseaddanyadditionalcommentsaboutthisexaminationthatwillassistusinplanningforfutureexaminations:

Documentyouractualnumberofcredits(hoursofstudy)forthisactivity:
Enteravalueof110
ACCREDITATION
TheAmericanAcademyofOrthopaedicSurgeonsisaccreditedbytheAccreditationCouncilfor
ContinuingMedicalEducationtoprovidecontinuingmedicaleducationforphysicians.

CMECREDIT
U.S.Physicians:TheAmericanAcademyofOrthopaedicSurgeonsdesignatesthiseducational
activityforamaximumof10AMAPRACategory1Credits.Physiciansshouldonlyclaimcredits
commensuratewiththeextentoftheirparticipationintheactivity.
InternationalPhysicians:TheAMAhasdeterminedthatphysiciansnotlicensedintheUnited
StatesbutwhoparticipateinthisCMEactivityareeligibleforAMAPRACategory1Credits.
AlliedHealthProfessionals:TheAcademyisnotaccreditedtooffercreditfornursesandotherallied
healthprofessionals.TodetermineifactivitiesofferingCategory1CMEcreditareacceptableforyour
licensingorcertificationneedspleasecontacttherelevantorganizationsdirectly.

EDUCATIONALOBJECTIVES
Differentiateamongtreatmentoptionsformanagingpatientswithpaininthefootandankle
Differentiateamongtreatmentoptionsforpatientswithdeformitiesofthefoot
Applyknowledgeoffootandankleanatomyindeterminingsurgicalexposuresforthe
forefoot,midfoot,hindfoot,andankle
Determinemethodstotreatmusculoskeletaltraumaofthefootandankle
Developtreatmentplansforpatientswithdiabetesmellitus,tendonandligamentruptures,
andneuropathiesofthefootandankle
Determinemethodstotreatsofttissueinjuriesinthefootandankle

ItisthegoaloftheAcademytopromotesafeandeffectiveorthopaediccareinallourprograms,
products,andservices.

Produced by the American Academy of Orthopaedic Surgeons

Foot and Ankle Evaluation Subcommittee of

the Evaluation Committee


ArthurK.Walling,MD,Chair
Director,FootandAnkleFellowship,FloridaOrthopaedicInstitute,Tampa,FL
BradleyJ.Brainard,MD
OrthopaedicSurgeon,PrivatePractice;ClinicalFaculty,UniversityofArizona,Tucson,AZ
MichaelP.Clare,MD
DirectorofFellowshipEducation,FootandAnkleFellowship,FloridaOrthopaedicInstitute,
Tampa,FL
KeithA.Heier,MD
OrthopaedicSurgeon,PrivatePractice,Dallas,TX
JohnV.Marymont,MD
AssociateProfessor,Chief,FootandAnkleSection,DepartmentofOrthopaedicSurgery,Baylor
CollegeofMedicine,Houston,TX
JamesD.Michelson,MD
Professor,OrthopaedicSurgery,DepartmentofOrthopaedicsandRehabilitation,Universityof
VermontCollegeofMedicine,Burlington,VT
JohnA.Papa,MD
ClinicalAssistantProfessor,DepartmentofSurgery,FloridaStateUniversityCollegeof
Medicine,Orlando,FL;OrthopaedicSurgeon,PrivatePractice,WinterPark,FL
MichaelS.Pinzur,MD
ProfessorofOrthopaedicSurgery,LoyolaUniversityHealthSystem,Maywood,IL
AndrewK.Sands,MD
SectionChief,FootandAnkleSurgery,DepartmentofOrthopaedicSurgery,Director,Footand
AnkleFellowship,SaintVincentsHospital,NewYork,NY
KennethA.Stephenson,MD
AssociateProfessor,TexasTechHealthSciencesCenter;PrivatePractice,Centerfor
OrthopaedicSurgery,Lubbock,TX
AnthonyD.Watson,MD
OrthopaedicSurgeon,PrivatePractice,Pittsburgh,PA

DisclosureInformationforMembersofCommittee:Foot&AnkleEvaluationSubcommittee
ArthurK.Walling,MD(Chair):4(AO);7(AO;DePuy,AJohnson&JohnsonCompany;DJ
Orthopaedics;EncoreMedical;Johnson&Johnson;LinkOrthopaedics;Smith&Nephew;Stryker;
Zimmer);8(Johnson&Johnson;Merck);10(LinkOrthopaedics).Submittedon:11/20/2007andlast
confirmedasaccurateon04/22/2008.
BradleyJ.Brainard,MD(Member):1(IamontheBoardofDirectorsoftheTucsonOrthopaedic
InstituteSurgeryCenter,whichisajointventurebetweenmypracticeandTucsonMedicalCenter);5A
(Biomimetic);7(Zimmer);8(Mytrustaccountishandledbymoneymanagers,andhealthcarestocksare
partofthatportfolio.Idon'thaveanyindividualholdings.).Submittedon:06/09/2008.
MichaelP.Clare,MD(Member):(n).Submittedon:11/14/2007andlastconfirmedasaccurateon
04/03/2008.
KeithA.Heier,MD(Member):5A(DePuy,AJohnson&JohnsonCompany).Submittedon:
04/23/2008.
JohnV.Marymont,MD(Member):(n).Submittedon:10/18/2007andlastconfirmedasaccurateon
04/23/2008.
JamesD.Michelson,MD(Member):(n).Submittedon:09/14/2007andlastconfirmedasaccurateon
04/03/2008.
JohnA.Papa,MD(Member):(n).Submittedon:11/20/2007andlastconfirmedasaccurateon
04/22/2008.
MichaelS.Pinzur,MD(Member):2(IamanAssociateEditorforFoot&AnkleInternational);4
(DePuy,AJohnson&JohnsonCompany;SBI;Smith&Nephew);5A(SBI;Smith&Nephew);7
(Synthes;DePuy,AJohnson&JohnsonCompany);10(none).Submittedon:06/09/2008.
AndrewK.Sands,MD(Member):1(AmericanOrthopaedicFootandAnkleSociety);2(Footand
AnkleInternational);4(Zimmer);5B(Synthes);6(Saunders/MosbyElsevier);7(AO;Biomet;
MedtronicSofamorDanek;Merck;Mitek;Philips;Stryker;Synthes;Zimmer);8(AmgenCo;Bristol
MyersSquibb;Pfizer);9(Saunders/MosbyElsevier);10(AO;Synthes;Zimmer).Submittedon:
11/30/2007.
KennethA.Stephenson,MD(Member):1(NorthstarSurgeryCenterLubbock,Texas);8(Orthohelix).
Submittedon:10/18/2007andlastconfirmedasaccurateon04/03/2008.
AnthonyD.Watson,MD(Member):(n).Submittedon:08/28/2007andlastconfirmedasaccurateon
10/07/2007.

DisclosureItems:(n)=RespondentansweredNotoallitemsindicatingnoconflicts.1=Board
member/owner/officer/committeeappointments;2=Medical/OrthopaedicPublications;3=Royalties;4=
Speakersbureau/paidpresentations;5A=Paidconsultant;5B=Unpaidconsultant;6=Researchor
institutionalsupportfromapublisher;7=Researchorinstitutionalsupportfromacompanyorsupplier;8=
StockorStockOptions;9=Otherfinancial/materialsupportfromapublisher;10=Otherfinancial/material
supportfromacompanyorsupplier.

AmericanAcademyofOrthopaedicSurgeons
MarkWieting,ChiefEducationOfficer
HowardMevis,Director,ElectronicMedia,Evaluation&CourseOperations
LauraHruska,ExaminationsManager
MarcieL.Lampert,SeniorExaminationsCoordinator

EditorialStaff
MarilynL.Fox,PhD,Director,Publications
GayleMurray,AssociateSeniorEditor
Disclaimer
ThematerialpresentedinthisspecialinterestexaminationhasbeenmadeavailablebytheAmerican
AcademyofOrthopaedicSurgeons foreducationalpurposesonly.Thismaterialisnotintendedtopresent
theonly,ornecessarilythebest,methodsorproceduresforthemedicalsituationsdiscussed,butratheris
intendedtorepresentanapproach,view,statement,oropinionoftheauthor(s)orproducer(s),whichmaybe
helpfultootherswhofacesimilarsituations.

SomedrugsormedicaldevicesdemonstratedinAcademycoursesordescribedinAcademyprintor
electronicpublicationshavenotbeenclearedbytheFoodandDrugAdministration(FDA)orhavebeen
clearedforspecificusesonly.TheFDAhasstatedthatitistheresponsibilityofthephysiciantodetermine
theFDAclearancestatusofeachdrugordeviceheorshewishestouseinclinicalpractice.

Atthetimeofthiswriting,bonescrewsplacedposteriorlyintovertebralelementshavebeenclearedforusein
thisspecificmannerbytheFoodandDrugAdministration(FDA)toprovideimmobilizationandstabilization
asanadjuncttofusioninthetreatmentofthefollowingacuteandchronicinstabilityordeformitiesofthe
thoracic,lumbar,andsacralspine:degenerativespondylolisthesiswithobjectiveevidenceofneurologic
impairment;fracture;dislocation;scoliosis;kyphosis;spinaltumor,andfailedpreviousfusion
(pseudarthrosis).Inaddition,anteriorvertebralbodyscrews(cervical,thoracic,andlumbar)areClassII
devicesandcanbeusedaslabeledinvertebralbodies.

Furthermore,anystatementsaboutcommercialproductsaresolelytheopinion(s)oftheauthor(s)anddonot
representanAcademyendorsementorevaluationoftheseproducts.Thesestatementsmaynotbeusedin
advertisingorforanycommercialpurpose.

Copyright 2009
bytheAmericanAcademyofOrthopaedicSurgeons
Allrightsreserved.NopartofFootandAnkleSelfAssessmentExaminationmaybereproduced,storedinaretrieval
system,ortransmittedinanyformorbyanymeans,electronic,mechanical,photocopying,recording,orotherwise
withoutthepriorwrittenpermissionofthepublisher.

Questions
1. Figure shows the radiograph of a 60-year-old woman who underwent
a previous operation for great toe pain 20 years ago. She has had
increasing pain over the past 5 years and now reports pain with any
motion, swelling, and clicking. She also reports pain under the ball of
foot. What is the most appropriate management to alleviate her
metatarsalgia and great toe pain?
1- Orthotic with Mortons extension
2- In situ great toe fusion with implant removal
3- Keller resection arthroplasty
4- Implant removal with structural bone grafting and
metatarsophalangeal (MTP) fusion
5- Implant removal, bone grafting, and titanium total toe implants

PREFERRED RESPONSE: 4

DISCUSSION: The patient has a failed Silastic implant. Nonsurgical management will not work at
this point. A Keller resection will only exacerbate her metatarsalgia. Implant removal with
structural bone grafting and MTP fusion is the most appropriate choice because restoration of length
is needed to alleviate the forefoot pain and bone grafting is required to fuse the MTP joint because
there is an abundance of osteolysis. Total toe implants do not offer good long-term outcomes and are
very difficult to fit into the large exploded-out cavity of the proximal phalanx.

REFERENCES: Hecht PJ, Gibbons MJ, Wapner KL, et al: Arthrodesis of the first
metatarsophalangeal joint to salvage failed silicone implant arthroplasty. Foot Ankle Int
1997;18:383-390.
Myerson MS, Schon LC, McGuigan FX, et al: Results of arthrodesis of the hallux
metatarsophalangeal joint using bone graft for restoration of length. Foot Ankle Int
2000;21:297-306.

2. A 47-year-old man with Charcot-Marie-Tooth (CMT) disease was treated with a fifth
metatarsal head resection for a symptomatic bunionette 2 years ago. What is the most likely
complication seen at this time?
1- Transfer lesion to the fourth metatarsal head
2- Fifth toe fixed claw toe deformity
3- Ulceration at the level of the resected head
4- Peroneal atrophy
5- Charcot arthropathy with midfoot collapse and forefoot abduction

PREFERRED RESPONSE: 1

DISCUSSION: CMT is characterized by a cavovarus foot position that increases weight-bearing


stresses along the lateral border. Removal of the fifth metatarsal head carries the risk of creating a
transfer lesion at the fourth metatarsal head, particularly with a cavovarus foot. Claw toes are
common in CMT, but the fifth toe would be flail in this situation. Ulceration is unlikely given the
lack of underlying bone. Peroneal atrophy is associated with CMT but would not be a complication
of this procedure. Charcot arthropathy is a neuropathic process frequently seen in individuals with
diabetes mellitus.

REFERENCES: Kitaoka HB, Holiday AD Jr: Metatarsal head resection for bunionette: Long-term
followup. Foot Ankle 1991;11:345-349.
Coughlin MJ, Mann RA, Saltzman CL: Surgery of the Foot and Ankle, ed 8. Philadelphia, PA,
Mosby, 2007, pp 1312-1315.

3. A 19-year-old man was struck by a car and is seen in the emergency department with a grade
IIIC open distal tibia and fibula fracture. Examination reveals that the toes are cool and
dusky with a sluggish capillary refill. Angiography reveals a lesion in the posterior tibial
artery amenable to repair. There is no sensation on the plantar aspect of the foot, and he is
unable to flex his toes. A clinical photograph and radiograph are shown in Figures a and b.
What is the next most appropriate step in management?
1- Irrigation and debridement with immediate intramedullary fixation, vascular repair,
and primary closure
2- Irrigation and debridement with external fixation, vascular repair, and delayed closure
3- Irrigation and debridement with external fixation, vascular repair, exploration of the
tibial nerve, and delayed closure
4- Guillotine amputation at the fracture site with delayed closure
5- Immediate below-knee amputation

PREFERRED RESPONSE: 2

DISCUSSION: In the past, loss of plantar sensation in this grade IIIC tibial fracture would have
been an indication for below-knee amputation regardless of the potential for vascular repair.
However the 2002 LEAP study divided 55 patients with loss of plantar sensation into two groups,
the insensate amputation group and the insensate limb salvage group, with 55% of patients in the
insensate salvage group regaining normal sensation 2 years after injury. Furthermore, those in the
salvage group who remained insensate after 2 years had equivalent outcomes to those in the
amputation group. Because of these findings, limb salvage with vascular repair and external
stabilization with delayed closure is deemed appropriate treatment. Immediate intramedullary
fixation is not indicated. Because ischemia, contusion, and stretch can adversely affect the tibial
nerve, the additional insult of exploration of the nerve is also not advisable given the soft-tissue
compromise.

REFERENCES: Bosse MJ, McCarthy ML, Jones AL, et al: The insensate foot following severe
lower extremity trauma: An indication for amputation? J Bone Joint Surg Am 2005;87:2601-2608.
Lange RH, Bach AW, Hansen ST Jr, et al: Open tibial fractures with associated vascular injuries:
Prognosis for limb salvage. J Trauma 1985;25:203-208.
Mackenzie EJ, Bosse MJ, Kellam JF, et al: Factors influencing the decision to amputate or
reconstruct after high-energy lower extremity trauma. J Trauma 2002;52:641-649.
Baumgaertner MR, Tornetta P III (eds): Orthopaedic Knowledge Update: Trauma 3. Rosemont, IL,
American Academy of Orthopaedic Surgeons, 2005, pp 65-74.

4. The pathophysiology of a claw toe deformity includes muscular imbalance caused by which
of the following relatively strong structures?

1- Intrinsics overpowering a relatively weak extensor digitorum longus


2- Plantar plate overpowering a relatively weak extensor digitorum brevis
3- Lumbricals overpowering relatively weak intrinsics
4- Extensor digitorum longus overpowering relatively weak intrinsics
5- Extensor digitorum longus overpowering a relatively weak extensor digitorum brevis

PREFERRED RESPONSE: 4

DISCUSSION: The dynamic forces acting to maintain the position of the proximal phalanx at the
head of the metatarsal are a balance between the extensor digitorum longus and the weaker intrinsic
muscles. With hyperextension at the metatarsophalangeal joint, the intrinsic muscles become less
efficient as plantar flexors. Consequently, the hyperextension deformity progresses in the
metatarsophalangeal joint as the opposition of the intrinsic muscles to the extensor tendon lessens.
This is in contrast to the situation in the interphalangeal joints, where the stronger flexors overpower
the weaker intrinsic muscles, which act as the extensors. This combination of events leads to
hyperextension at the metatarsophalangeal joint and flexion deformities at the interphalangeal joints,
resulting in claw toe.

REFERENCES: Mizel MS, Yodlowski ML: Disorders of the lesser metatarsophalangeal Joints.
J Am Acad Orthop Surg 1995;3:166-173.
Coughlin MJ, Mann RA: Surgery of the Foot and Ankle, ed. 7. St Louis, MO, Mosby, 1999,
pp 325-328.

5. A 26-year-old woman is seen in the emergency department with an intra-articular distal tibia
fracture and a fibular fracture (pilon). The patient, her husband, and three small children have
recently immigrated to the United States from Mexico. The husband and wife have both been
in a migrant labor camp but have no immediate relatives in the States. What factor is most
important when considering her recommended care and treatment?

1- It may be difficult to obtain informed consent, even with an interpreter.


2- The husband may be unwilling to allow his wife to have the appropriate surgery.
3- Associated comorbidities in the patient increase the risk of surgical intervention.
4- There is the potential of not being able to obtain a satisfactory CT scan because
of claustrophobia.
5- Postsurgical care may be jeopardized by the patients role as caretaker for her
children, thus compromising her ability to comply with weight-bearing restrictions.

PREFERRED RESPONSE: 5

DISCUSSION: With documented use of a competent interpreter, informed consent should not be an
issue. In Hispanic families, the husband often makes the ultimate decision regarding proceeding with
surgery; however, he would not be expected to withhold recommended treatment. Hispanics may
have a higher risk of comorbidities, but you do not expect this to be a significant concern with this
patient. Claustrophobia and some fear of the unfamiliar may make additional imaging studies more
difficult to arrange, but not impossible. The real concern is that with no extended family and three
small children, the postoperative demand on the patient could significantly jeopardize her ability to
comply with weight-bearing restrictions and overall ambulatory demands. Discharge planning and
appropriate help may be paramount for a good outcome.

REFERENCES: Jimenez R, Lewis VO (eds): Culturally Competent Care Guidebook. Rosemont, IL,
American Academy of Orthopaedic Surgeons, 2007.

6. A 57-year-old man with type II diabetes mellitus was successfully treated for a first
occurrence forefoot full-thickness (Wagner II) diabetic foot ulcer underlying the third
metatarsal head with associated hammertoe with a series of weight-bearing total contact
casts. There was no evidence of osteomyelitis. The ulcer is now fully healed. He is
insensate to the Semmes-Weinstein 5.07 (10 gm) monofilament. What is the next most
appropriate step in management?

1- No further treatment is advised unless a second ulcer develops


2- Oxford shoes with a rubber sole
3- Depth-inlay shoes with a custom accommodative foot orthosis
4- Dorsiflexion third metatarsal osteotomy
5- Achilles tendon lengthening

PREFERRED RESPONSE: 3

DISCUSSION: This is the first occurrence of diabetic foot-specific morbidity. The patient has a foot
deformity, a history of a diabetic foot ulcer, and is insensate to the monofilament. He is at moderate
risk for the development of a recurrent ulcer. This is best avoided with therapeutic footwear.
Commercially available depth-inlay shoes should be combined with a custom accommodative foot
orthosis to accommodative the deformity.

REFERENCES: Pinzur MS, Slovenkai MP, Trepman E, et al: Guidelines for diabetic foot care:
Recommendations endorsed by the Diabetes Committee of the American Orthopaedic Foot and
Ankle Society. Foot Ankle Int 2005;26:113-119.
Pinzur MS, Dart HC: Pedorthic management of the diabetic foot. Foot Ankle Clin 2001;6:205-214.
7. A 28-year-old man has had a 2-year history of progressive lateral ankle pain. History reveals
that he underwent a triple arthrodesis at age 13 for a tarsal coalition. The pain has been
refractory to braces, custom inserts, and nonsteroidal anti-inflammatory drugs. Weight-
bearing radiographs of the ankle and foot are shown in Figures a through d. Surgical
management should include which of the following?

1- Ankle arthroscopy and lateral ligament reconstruction


2- Tendon transfer, lateral column lengthening, and heel cord lengthening
3- Ankle arthrodesis with retrograde intramedullary nail fixation
4- Calcaneal osteotomy and transverse tarsal osteotomy
5- Total ankle arthroplasty and deltoid ligament reefing

PREFERRED RESPONSE: 4

DISCUSSION: The patient has a valgus-supination triple arthrodesis malunion. Weight-bearing


radiographs show excessive residual valgus through the subtalar joint, producing lateral subfibular
impingement, and residual forefoot abduction and midfoot supination through the talonavicular joint,
lateralizing the weight-bearing forces through the foot. The deformity is best managed with a medial
displacement calcaneal osteotomy and transverse tarsal derotational osteotomy. Ankle arthroscopy
and lateral ligament reconstruction are indicated in the event of ligament instability. Tendon transfer,
lateral column lengthening, and heel cord lengthening are used for treatment of adult flatfoot from
posterior tibial tendon insufficiency. Ankle arthrodesis and ankle arthroplasty are not indicated in
this patient because the lateral ankle symptoms are the result of the underlying deformity in the
hindfoot, the patient is young, and the ankle joint is relatively normal.

REFERENCES: Haddad SL, Myerson MS, Pell RF IV: Clinical and radiographic outcome of
revision surgery for failed triple arthrodesis. Foot Ankle Int 1997;18:489-499.
Menp H, Lehto MU, Belt EA: What went wrong in triple arthrodesis? An analysis of failures in
21 patients. Clin Orthop Relat Res 2001;391:218-223.
8. If heel varus corrects with a Coleman block test, then the hindfoot deformity is flexible. This
test proves that the varus is due to a

1- dorsiflexed first ray.


2- varus position of the forefoot.
3- plantar flexed first ray.
4- valgus hindfoot.
5- rigid flatfoot.

PREFERRED RESPONSE: 3

DISCUSSION: The Coleman block test is used to evaluate the effect of the forefoot on the rearfoot
varus. If the deformity corrects with the block, then the hindfoot deformity is flexible and the varus
position is secondary to the plantar flexed first ray or valgus position of the forefoot. A rearfoot
orthotic will not correct the forefoot cause of the deformity. The patient still may need a lateralizing
calcaneal osteotomy to realign the hindfoot.

REFERENCES: Younger AS, Hansen ST Jr: Adult cavovarus foot. J Am Acad Orthop Surg
2005;13:302-315.
Alexander IJ, Johnson KA: Assessment and management of pes cavus in Charcot-Marie-Tooth
disease. Clin Orthop Relat Res 1989;246:273-281.

9. A 27-year-old man now reports dorsiflexion and inversion weakness after an automobile
collision 6 months ago in which compartment syndrome developed isolated to the anterior
and deep posterior compartments. Examination reveals the development of a progressive
cavovarus deformity, but the ankle and hindfoot remain flexible. In addition to Achilles
tendon lengthening, which of the following procedures is most likely to improve the motor
balance of his foot and ankle?

1- Anterior tibialis tendon transfer to the dorsolateral midfoot


2- Posterior tibialis tendon transfer to the dorsolateral midfoot
3- Peroneus longus tendon transfer to the dorsolateral midfoot
4- Peroneus brevis tendon transfer to the dorsolateral midfoot
5- Flexor hallucis longus tendon transfer to the peroneus brevis

PREFERRED RESPONSE: 3

DISCUSSION: Compartment syndrome of the anterior and deep posterior compartments results in
anterior tibialis and posterior tibialis tendon weakness, respectively. Furthermore, the long flexors to
the hallux and lesser toes will be weak as well. The intact peroneus longus overpowers the weak
anterior tibialis tendon, resulting in plantar flexion of the first metatarsal, cavus, and hindfoot varus.
Therefore, transferring the peroneus longus to the dorsolateral midfoot reduces the first metatarsal
plantar flexion torque, and possibly augments ankle dorsiflexion torque.

REFERENCES: Hansen ST: Functional Reconstruction of the Foot and Ankle. Philadelphia, PA,
Lippincott, Williams & Wilkins, 2000, pp 433-435.
Vienne P, Schoniger R, Helmy N, et al: Hindfoot instability in cavovarus deformity: Static and
dynamic balancing. Foot Ankle Int 2007;28:96-102.

10. Figures a through c show the radiographs of a 43-year-old woman who sustained a twisting
injury to her right ankle. She has ankle pain and tenderness medially and laterally. To help
determine the optimal treatment, an external rotation stress radiograph of the ankle is
obtained. This test is designed to evaluate the integrity of what structure?

1- Posterior talofibular ligament


2- Distal tibiofibular syndesmosis
3- Anterior talofibular ligament
4- Deltoid ligament
5- Calcaneofibular ligament

PREFERRED RESPONSE: 4

DISCUSSION: In the presence of a supination external rotation-type fracture of the distal fibula
(Weber type B), stability of the ankle is best assessed by performing an external rotation stress AP
view of the ankle. This test is used to assess the integrity of the deltoid ligament. The presence of a
deltoid ligament rupture results in instability and generally is best managed surgically. The gravity
stress test can also be used.

REFERENCES: Egol KA, Amirtharajah M, Tejwani NC, et al: Ankle stress test for predicting the
need for surgical fixation of isolated fibular fractures. J Bone Joint Surg Am 2004;86:2393-2398.
McConnell T, Creevy W, Tornetta P III: Stress examination of supination external rotation-type
fibular fractures. J Bone Joint Surg Am 2004;86:2171-2178.
Schock HJ, Pinzur M, Manion L, et al: The use of the gravity or manual-stress radiographs in the
assessment of supination-external rotation fractures of the ankle. J Bone Joint Surg Br
2007;89:1055-1059.

11. A 29-year-old patient sustains a closed, displaced joint depression intra-articular calcaneus
fracture. In discussing potential complications of surgical intervention through an extensile
lateral approach, which of the following is considered the most common complication
following surgery?

1- Nonunion
2- Deep infection
3- Delayed wound healing
4- Peroneal tendinitis
5- Posttraumatic arthritis

PREFERRED RESPONSE: 3

DISCUSSION: Delayed wound healing and wound dehiscence is the most common complication of
surgical management of calcaneal fractures through an extensile lateral approach, occurring in up to
25% of patients. Most wounds ultimately heal with local treatment; the deep infection rate is
approximately 1% to 4% in closed fractures. Posttraumatic arthritis may develop despite open
reduction and internal fixation, but the percentages remain low. Peroneal tendinitis may occur from
adhesions within the tendon sheath or from prominent hardware but is relatively uncommon.
Nonunion of a calcaneal fracture is rare.

REFERENCES: Sanders RW, Clare MP: Fractures of the calcaneus, in Coughlin MJ, Mann RA,
Saltzman CL (eds): Surgery of the Foot and Ankle, ed 8. Philadelphia, PA, Mosby-Elsevier, 2007,
vol 2, pp 2017-2073.
Sanders RW, Clare MP: Fractures of the calcaneus, in Bucholz RW, Heckman JD,
Court-Brown C (eds): Rockwood and Greens Fractures in Adults, ed 6. Philadelphia, PA,
Lippincott Williams & Wilkins, 2006, vol 2, pp 2293-2336.
12. Figures a and b show the radiographs of a 56-year-old man who was seen in the emergency
department following a twisting injury to his left ankle. Examination in your office 3 days
later reveals marked swelling and diffuse tenderness to palpation about the ankle and leg.
What is the next most appropriate step in management?

1- MRI of the ankle


2- CT of the ankle
3- Technetium bone scan
4- Radiographs of the tibia and fibula
5- Repeat radiographs of the ankle in 5 to 7 days

PREFERRED RESPONSE: 4

DISCUSSION: The radiographs show an isolated posterior malleolus fracture which, given the
injury mechanism, is highly suspicious for a Maisonneuve injury. As with any suspected extremity
injury, radiographs including the joints above and below the level of injury are acutely indicated.
Although MRI may reveal a ligamentous injury to the ankle and CT may show asymmetry of the
ankle mortise or syndesmosis, both studies are considerably more costly and are not indicated in the
absence of a complete radiographic work-up. Technetium bone scan is nonspecific and would be of
limited value in this instance, as would repeat radiographs of the ankle.

REFERENCES: Walling AK, Sanders RW: Ankle fractures, in Coughlin MJ, Mann RA, Saltzman
CL (eds): Surgery of the Foot and Ankle, ed 8. Philadelphia, PA, Mosby-Elsevier, 2007, vol 2,
pp 1973-2016.
Marsh JL, Saltzman CL: Ankle fractures, in Bucholz RW, Heckman JD, Court-Brown C (eds):
Rockwood and Greens Fractures in Adults, ed 6. Philadelphia, PA, Lippincott Williams & Wilkins,
2006, vol 2, pp 2147-2247.
13. A 61-year-old man has a symptomatic bunionette that is
refractory to nonsurgical management. A radiograph is
shown in Figure What is the optimal surgical correction?

1- Fifth metatarsal head lateral ostectomy


2- Fifth metatarsal head excision
3- Metatarsal osteotomy and fifth metatarsal head
ostectomy
4- Fifth metatarsal plantar condylectomy
5- Fifth metatarsophalangeal Silastic implant
arthroplasty

PREFERRED RESPONSE: 3

DISCUSSION: The patient has a bunionette with a large 4-5 intermetatarsal angle. This requires not
only ostectomy of the lateral prominence but metatarsal osteotomy to decrease the intermetatarsal
angle. Excising the head results in a flail joint and creates the possibility of a transfer lesion.
Condylectomy can reduce plantar pressures but does not address the bunionette. The joint surface is
well maintained, thus there are no indications for resection.

REFERENCES: Coughlin MJ: Treatment of bunionette deformity with longitudinal diaphyseal


osteotomy with distal soft tissue repair. Foot Ankle 1991;11:195-203.
Koti M, Maffulli N: Bunionette. J Bone Joint Surg Am 2001;83:1076-1082.
14. A 25-year-old woman with a healed proximal tibiofibular fracture treated with an
intramedullary nail 2 years ago is currently wearing an ankle-foot orthosis (AFO) and reports
a persistent foot drop. She is unhappy with the AFO and has not seen any functional
improvement despite months of physical therapy. Serial electromyograms (EMG) show no
recent change over the past year. Examination and EMG findings are consistent with a
tibialis anterior 1/5, extensor hallucis longus 2/5, extensor digitorum longus 2/5, posterior
tibial tendon (PTT) 5/5, peroneals 3/5, flexor hallucis longus 5/5, and gastrocsoleus 5/5. No
discrete nerve lesion was identified. The patient has a flexible equinovarus contracture.
What is the most appropriate management?

1- Continued AFO bracing and therapy


2- Ankle fusion
3- Exploration and release of the common peroneal nerve
4- Transfer of the PTT through the interosseous membrane with attachment to the
tibialis anterior and peroneus tertius above the level of the ankle, debridement of the
anterior compartment, and Achilles tendon lengthening
5- Transfer of the peroneus longus to the dorsum of the foot and Achilles tendon
lengthening

PREFERRED RESPONSE: 4

DISCUSSION: This pattern of injury is consistent with an unrecognized compartment syndrome of


the anterior and lateral compartments. Transfer of the PTT through a long incision in the
interosseous membrane corrects the foot drop deformity, and allows adequate dorsiflexion provided
that the tendon to be transferred has a strength of 5/5. Muscles/tendons typically lose one grade of
strength after transfer. Transfer into the tendons at the level of the ankle prevents overtensioning or
pullout of a PTT tendon that is not long enough. Debridement of the scarred muscle in the anterior
compartment decreases the risk of scarring down to the tendon transfer. Transfer of the peroneus
longus is not preferred given its relative lack of strength and line of pull. Continued therapy and
bracing are unlikely to lead to further improvement at 2 years after injury. An ankle fusion would
correct the foot drop but would not address the tendon imbalances between the tibialis anterior and
the peroneus longus, and the PTT and the peroneus brevis.

REFERENCES: Hansen ST Jr: Functional Reconstruction of the Foot and Ankle. Philadelphia, PA,
Lippincott Williams & Wilkins, 2000, p 192.
Atesalp AS, Yildiz C, Komurcu M, et al: Posterior tibial tendon transfer and tendo-Achilles
lengthening for equinovarus foot deformity due to severe crush injury. Foot Ankle Int
2002;23:1103-1106.
Scott AC, Scarborough N: The use of dynamic EMG in predicting the outcome of split posterior
tibial tendon transfers in spastic hemiplegia. J Pediatr Orthop 2006;26:777-780.
Williams PF: Restoration of muscle balance of the foot by transfer of the tibialis posterior. J Bone
Joint Surg Br 1976;58:217-219.
15. When using a two-incision approach for open reduction and internal fixation of a Hawkins III
talar fracture-dislocation involving the talar neck and body, what anatomic structure must be
preserved to optimize outcome?

1- Deltoid branch of the artery of the tarsal canal


2- Dorsalis pedis artery
3- Tarsal sinus artery
4- Perforating peroneal artery
5- Navicular artery

PREFERRED RESPONSE: 1

DISCUSSION: A Hawkins III fracture-dislocation generally presents with posteromedial


displacement with the deltoid ligament intact. Therefore, the only remaining blood supply is the
deltoid branch of the artery of the tarsal canal originating from the posterior tibial artery. Often, the
medial malleolus is fractured, assisting in reduction and visualization of fracture reduction. If the
medial malleolus is intact, a medial malleolus osteotomy allows visualization of the reduction
without compromising the last remaining blood supply to the talus.

REFERENCES: Mulfinger GL, Trueta J: The blood supply of the talus. J Bone Joint Surg Br
1970;52:160-167.
Vallier HA, Nork SE, Barei DP, et al: Talar neck fractures: Results and outcomes. J Bone Joint Surg
Am 2004;86:1616-1624.

16. A 10-year-old boy who is active in soccer has had activity-related heel pain for the past 3
months. Examination reveals tenderness over the posterior heel and a tight Achilles tendon.
Radiographs demonstrate a 2-cm cyst in the anterior body of the calcaneus. His physes have
not closed. Based on these findings, what is the most appropriate management?

1- Bone scan
2- Curettage and bone grafting of the cyst
3- Cast immobilization
4- Observation with reduced activity
5- Skeletal survey

PREFERRED RESPONSE: 4

DISCUSSION: The most likely diagnosis is Severs disease, which is considered either an
apophysitis or a para-apophyseal stress fracture. It is common in athletic children and is associated
with a tight Achilles tendon. Cast immobilization may be necessary if activity reduction fails.
Calcaneal cysts are quite common and do not require any further diagnostic testing or treatment
unless they occupy the full width of the calcaneus or one third of the length of the calcaneus.

REFERENCES: Ogden JA, Ganey TM, Hill JD, et al: Severs injury: A stress fracture of the
immature calcaneal metaphysis. J Ped Orthop 2004;24:488-492.
Pogoda P, Priemel M, Linhart W, et al: Clinical relevance of calcaneal bone cysts: A study of 50
cysts in 47 patients. Clin Orthop Relat Res 2004;424:202-210.

17. A 35-year-old woman states that she stepped on a


piece of glass 6 months ago and reports numbness and
shooting pain along the plantar lateral forefoot. She had
previously received steroid injections in the 3 to 4
webspace. Examination reveals mild tenderness along
the plantar fascia; no Tinels sign is noted plantar
medially and no Mulders click is noted distally. An
MRI scan is shown in Figure What is the most likely
cause of the numbness?

1- Residual foreign body


2- Lateral plantar nerve laceration
3- Impingement of Baxters nerve
4- Interdigital neuroma
5- Digital nerve laceration

PREFERRED RESPONSE: 2

DISCUSSION: The MRI scan reveals a laceration through the abductor hallucis musculature and
lateral plantar nerve, producing numbness along its distribution. There is no evidence of a foreign
body on the MRI scan. Baxters nerve, or nerve to the abductor digiti quinti muscle, is the first
branch off the lateral plantar nerve and impingement of this nerve typically produces a Tinels sign
along the nerve branch deep to the abductor hallucis muscle. Interdigital neuroma would be
suggested by the presence of a Mulders click. A digital nerve laceration would exhibit isolated
numbness more distally.

REFERENCES: Baxter DE, Pfeffer GB: Treatment of chronic heel pain by surgical release of the
first branch of the lateral plantar nerve. Clin Orthop Relat Res 1992;279:229-236.
Sarrafian SK: Anatomy of the Foot and Ankle: Descriptive, Topographic, Functional. Philadelphia,
PA, JB Lippincott, 1983.
18. A 69-year-old man reports pain over his bunion while wearing shoes and pain in the joint
with push-off when barefoot. Nonsurgical management has failed to provide relief.
Radiographs are shown in Figures a and b. What is the surgical procedure of choice?

1- First metatarsophalangeal arthrodesis


2- Distal chevron osteotomy and bunionectomy with closing wedge osteotomy and soft-
tissue release
3- Bunionectomy with proximal metatarsal osteotomy
4- Bunionectomy with first metatarsal cuneiform fusion
5- Bunionectomy with proximal phalanx osteotomy and distal chevron osteotomy

PREFERRED RESPONSE: 1

DISCUSSION: Arthrodesis is indicated for severe bunion and hallux valgus deformities, but
particularly with extensive degenerative disease of the first metatarsophalangeal joint. The other
bunionectomy procedures have different indications, none of which include symptomatic first
metatarsophalangeal degenerative disease.

REFERENCES: Richardson EG (ed): Orthopaedic Knowledge Update: Foot and Ankle 3.


Rosemont, IL, American Academy of Orthopaedic Surgeons, 2004, pp 3-15.
Tourne Y, Saragaglia D, Zattara A, et al: Hallux valgus in the elderly: Metatarsophalangeal
arthrodesis of the first ray. Foot Ankle Int 1997;18:195-198.
19. A 65-year-old man has chronic Achilles insertional
tendinitis that is refractory to nonsurgical
management. A radiograph is shown in Figure.
Preoperative counseling should include a discussion
of the realistic duration of postoperative recovery.
You should inform the patient that his expected
recovery will last

1- 6 weeks.
2- 12 weeks.
3- 3 to 6 months.
4- 9 months.
5- 12 months.

PREFERRED RESPONSE: 5

DISCUSSION: An older patient with calcaneal enthesopathy may take a year or more to recover
after tendon debridement and calcaneal ostectomy. Young patients, and those with purely tendon
pathology, may recover more quickly.

REFERENCES: McGarvey WC, Palumbo RC, Baxter DE, et al: Insertional Achilles tendinitis:
Surgical treatment through a central tendon splitting approach. Foot Ankle Int 2002;23:19-25.
Watson AD, Anderson RB, Davis WH: Comparison of results of retrocalcaneal decompression for
retrocalcaneal bursitis and insertional Achilles tendinosis with calcific spur. Foot Ankle Int
2000;21:638-642.

20. Figures a and b show the clinical photograph and MRI scan of a plantar foot lesion.
If excisional biopsy is performed, what is the most likely complication?
1- Metastatic spread
2- Recurrence of the lesion
3- High likelihood of wound breakdown
4- Injury to the medial plantar nerve
5- Loss of the local windlass effect of the plantar fascia

PREFERRED RESPONSE: 2

DISCUSSION: The MRI scan shows plantar fibromatosis. The treatment is usually nonsurgical.
If surgery is indicated, wide local excision with excision of the entire plantar fascia is usually
indicated. The main problem with simple excision of the lesion is the high chance of recurrence.
The other listed complications are those that are a result of the wide local excision.

REFERENCES: Aluisio FV, Mair SD, Hall RL: Plantar fibromatosis: Treatment of primary and
recurrent lesions and factors associated with recurrence. Foot Ankle Int 1996;17:672-678.
Bos GD, Esther RJ, Woll TS: Foot tumors: Diagnosis and treatment. J Am Acad Orthop Surg
2002;10:259-270.

21. A patient with rheumatoid arthritis with both ankle and subtalar involvement was treated as
shown in Figures a and b. What complication is unique to this type of fixation?

1- Late rotatory deformity


2- Limb-length discrepancy
3- Talar osteonecrosis
4- Tibial stress fracture
5- Hardware failure
PREFERRED RESPONSE: 4

DISCUSSION: The interlocking screws at the proximal end of the rod can act as a stress riser and
lead to fracture. Postoperative pain at this level should prompt inclusion of this diagnosis in the
differential. Removing the screws following bone union can decrease the chances of this occurring.
A short rod that avoids the diaphyseal area may also be beneficial. Rotatory deformity is controlled
by the perpendicularly oriented distal transfixion screws. Talar osteonecrosis would be unusual
since the dissection can be minimized with an intramedullary rod. Any type of hardware can fail if
the construct does not lead to a solid arthrodesis.

REFERENCES: Nunley JA, Pfeffer GB, Sanders RW, et al (eds): Advanced Reconstruction: Foot
and Ankle. Rosemont, IL, American Academy of Orthopaedic Surgeons, 2004, pp 236-237.
Thordarson DB, Chang D: Stress fractures and tibial cortical hypertrophy after tibiotalocalcaneal
arthrodesis with an intramedullary nail. Foot Ankle Int 1999;20:497-500.
Hammett R, Hepple S, Forster B, et al: Tibiotalocalcaneal (hindfoot) arthrodesis by retrograde
intramedullary nailing using a curved locking nail: The results of 52 procedures. Foot Ankle Int
2005;26:810-815

22. A 68-year-old man fell off a 20-foot mountain cliff and


was seen in the emergency department the following
morning. A radiograph is shown in Figure. He is a
nonsmoker with medical comorbidities of
hypertension and hypercholesterolemia that is well
controlled with medicine and diet. Capillary refill and
sensation are intact distally and the patient is able to
move his toes with mild discomfort. Serosanguinous
fracture blisters are present laterally, and the foot is
swollen and red. What is the most appropriate
management?

1- Short leg cast for 6 weeks


2- Splinting with early range of motion at 3 weeks
3- Immediate open reduction and internal fixation through a medial approach
4- Delayed open reduction and internal fixation
5- Fusion

PREFERRED RESPONSE: 4

DISCUSSION: Whereas a patient age of older than 50 years used to be a contraindication for open
reduction and internal fixation of displaced intra-articular calcaneal fractures, new data suggest that
the presence of associated medical comorbidities that affect wound healing such as smoking,
diabetes mellitus, and peripheral vascular disease are more relevant to postoperative functional
outcome. Surgical treatment of Sanders II and III displaced intra-articular calcaneal fractures with
initial Bohler angles of > 15 degrees results in better outcomes as compared to nonsurgical
management. Indications for primary fusion might include Sanders IV fractures in which articular
congruity or Bohler angles cannot be restored. Given the condition of the soft tissues at
presentation, delayed fixation is recommended.

REFERENCES: Herscovici D Jr, Widmaier J, Scaduto JM, et al: Operative treatment of calcaneal
fractures in elderly patients. J Bone Joint Surg Am 2005;87:1260-1264.
Buckley R, Tough S, McCormack R, et al: Operative compared with nonoperative treatment of
displaced intra-articular calcaneal fractures: A prospective, randomized, controlled multicenter trial.
J Bone Joint Surg Am 2002;84:1733-1744.
Csizy M, Buckley R, Tough S, et al: Displaced intra-articular calcaneal fractures: Variables
predicting late subtalar fusion. J Orthop Trauma 2003;17:106-112.

23. A 45-year-old woman has had intense pain in her foot for the
last 3 days. She also reports a mild fever and difficulty with shoe
wear. Examination reveals a swollen, slightly erythematous warm
foot with tenderness at the great toe metatarsophalangeal joint and
pain with passive motion of the joint. An AP radiograph is shown in
Figure Which of the following will best aid in determining a
definitive diagnosis?

1- Gadolinium-enhanced MRI of the great toe


2- Serum uric acid level, C-reactive protein, and
erythrocyte sedimentation rate
3- Serum rheumatoid factor
4- Aspiration of the first metatarsophalangeal joint
5- Fasting serum glucose level

PREFERRED RESPONSE: 4

DISCUSSION: The patient has gouty arthropathy of the first metatarsophalangeal joint. This
definitive diagnosis is achieved with aspiration of the joint and polarized light microscopy that
shows needle-shaped negatively birefringent monosodium urate crystals. Differential diagnoses of
infectious arthritis and pseudogout are also definitively made through joint aspiration. Although
rheumatoid arthritis is a possibility, a serum rheumatoid factor is not always diagnostic and a patient
with rheumatoid arthritis may have concomitant gouty arthritis. The radiographic findings are not
typical of diabetes mellitus or of a patient with Charcot arthropathy.

REFERENCES: Wise CM, Agudelo CA: Diagnosis and management of complicated gout. Bull
Rheum Dis 1998;47:2-5.
Harris MD, Siegel LB, Alloway JA: Gout and hyperuricemia. Am Fam Physician 1999;59:925-934.
24. Figures a and b show the clinical photographs of a patient who was stranded in a subzero
region for several days. The photographs were taken the morning after arrival in the hospital.
The patient is otherwise healthy and fit, and takes no medication. He has no clinical signs of
sepsis. He reports burning pain and tingling in both feet. What is the
best treatment?

1- Moist dressings and continued observation


2- Debridement of the necrotic tissue
3- Amputation at the metatarsophalangeal level with open wound management
4- Closed forefoot amputation
5- Guillotine transtibial amputation

PREFERRED RESPONSE: 1

DISCUSSION: The patient has no clinical or observed signs of sepsis. The skin just proximal to the
gangrenous tissue appears somewhat hyperemic and is clearly viable. These wounds should be
managed much like burn wounds. Moist dressings should be used until the tissue clearly
demarcates. Much of the insult may simply be superficial and only require late debridement.

REFERENCES: McAdams TR, Swenson DR, Miller RA: Frostbite: An orthopedic perspective.
Am J Orthop 1999;28:21-26.
Taylor MS: Cold weather injuries during peacetime military training. Milit Med 1992;157:602-604.
25. The peroneus tertius is a commonly used landmark for arthroscopic portal placement.
What is the function of this tendon?

1- Dorsiflexion
2- Eversion
3- Dorsiflexion and eversion
4- Fifth toe extension
5- Lesser toe extension

PREFERRED RESPONSE: 3

DISCUSSION: The peroneus tertius, although absent in 10% of the population, originates on the
distal third of the extensor surface of the fibula and inserts onto the base of the fifth metatarsal,
possibly extending to the fascia over the fourth interosseous space. The muscle is located in the
anterior compartment of the leg and is innervated by the deep peroneal nerve. The tendon produces
dorsiflexion and eversion when walking and can be used as an insertion point during tendon
transfers to assist dorsiflexion. This tendon is peculiar to humans and is a proximally migrated deep
extensor of the fifth toe.

REFERENCES: Joshi SD, Joshi SS, Athavale SA: Morphology of the peroneus tertius muscle.
Clin Anat 2006;19:611-614.
Williams PL, Bannister LH, Berry MM, et al (eds): Grays Anatomy, ed 38. London, Churchill
Livingston, 1995, p 883.
Hansen ST Jr: Functional Reconstruction of the Foot and Ankle. Philadelphia, PA, Lippincott
Williams & Wilkins, 2000, p 21.

26. A 45-year-old man is seen in the emergency


department after returning from a 2-hour airplane
flight. He is reporting severe pain in his right leg
but has no trouble moving his ankle, leg, or knee.
Venous doppler testing reveals no evidence of
deep venous thrombosis. He is placed on IV
cephazolin but continues to worsen. On the third
day in the hospital he has increased pain, some
respiratory distress, and trouble maintaining his
blood pressure. His leg takes on the appearance
seen in Figure. An urgent MRI scan shows
thickening of the subcutaneous tissues and
superficial swelling in the leg but no evidence of an abscess. What is the next most
appropriate step in management?

1- Triple antibiotic coverage


2- Transfer to the ICU and a consult with infectious disease
3- Urgent irrigation and debridement with gentle skin closure
4- Urgent hyperbaric oxygen treatments and immunoglobulin
5- Urgent aggressive debridement of skin, subcutaneous fat, and fascia

PREFERRED RESPONSE: 5

DISCUSSION: The patient has necrotizing fasciitis, a rare and sometimes fatal disease that has
many different etiologies. Signs that this is not a normal infection are the worsening clinical
symptoms despite IV antibiotics and the systemic symptoms. He needs urgent surgical care before
he becomes completely septic and unstable. He needs very aggressive debridement of his tissues.
Hyperbaric oxygen and immunoglobulins are only anecdotally helpful, and would only be used
after surgery.

REFERENCES: Fontes RA, Ogilvie CM, Miclau T: Necrotizing soft-tissue infections. J Am Acad
Orthop Surg 2000;8:151-158.
Ozalay M, Ozkoc G, Akpinar S, et al: Necrotizing soft-tissue infection of a limb: Clinical
presentation and factors related to mortality. Foot Ankle Int 2006;27:598-605.

27. A 17-year-old girl with Charcot-Marie-Tooth disease reports the development of progressive
instability when walking on uneven surfaces. Her involved heel is positioned in varus when
viewed from behind. Examination reveals that she walks on the outer border of the involved
foot. She has full passive motion of the ankle and hindfoot joints. She is able to dorsiflex
the ankle against resistance. The heel varus fully corrects with the Coleman block test.
Standing radiographs reveal a cavus deformity with valgus of the forefoot. She would like to
avoid using an ankle-foot orthosis. What is the best surgical option?

1- Dorsiflexion osteotomy of the first metatarsal


2- Dorsiflexion osteotomy of the first metatarsal combined with anterior transfer of the
tibialis posterior
3- Triplanar osteotomy at the apex of the deformity
4- Triplanar osteotomy at the apex of the deformity combined with valgus calcaneal
osteotomy
5- Triplanar osteotomy at the apex of the deformity combined with anterior transfer of
the tibialis posterior

PREFERRED RESPONSE: 1

DISCUSSION: This deformity is early in the disease process. The foot is still flexible, as evidenced
by correction with the Coleman block test. A simple dorsiflexion osteotomy of the first metatarsal
should provide a plantigrade foot. More complex osteotomies are required later in the disease
process when the foot is not flexible and the deformity does not correct with the Coleman block test.
The patient may also require a tibialis anterior transfer later in the disease process but not at the
present time.
REFERENCES: Richardson EG (ed): Orthopaedic Knowledge Upate: Foot and Ankle 3. Rosemont,
IL, American Academy of Orthopaedic Surgeons, 2004, pp 135-144.
Dehne R: Congenital and acquired neurologic disorders, in Coughlan MJ, Mann RA (eds): Surgery
of the Foot and Ankle, ed 7. St Louis, MO, Mosby, 1999, vol 1, pp 525-557.

28. A 58-year-old man with type 1 diabetes mellitus is seen in the emergency department and he
reports a 3-day history of a red swollen foot but no history of trauma. Examination reveals
that the skin is intact, and the patient has discomfort with passive range of motion at the
ankle, hindfoot, and midfoot joints. He denies any fever. Laboratory studies show a WBC
count of 7,800/mm3, an erythrocyte sedimentation rate of 40 mm/h, a C-reactive protein level
of 23, and a serum glucose of 100. A radiograph and MRI scans are shown in Figures a
through c. What is the next most appropriate step in management?

1- Technetium Tc 99m triple phase bone scan


2- Admit for elevation, observation, and IV antibiotics
3- Total contact casting, no weight bearing, and weekly cast changes until erythema and
swelling is decreased
4- Immediate stabilization of the talus and midfoot with internal fixation and
postoperative splinting, no weight bearing, and total contact casting
5- Total contact casting, weight bearing as tolerated, with weekly cast changes

PREFERRED RESPONSE: 3

DISCUSSION: Whereas it is difficult to distinguish between cellulitis, septic joint, osteomyelitis,


and early Eichenholtz stage 1 Charcot, the presence of a fracture in the absence of ulcerations with a
normal WBC count and serum glucose strongly indicates that the described symptoms are due to an
early Charcot process alone. A technetium Tc 99m scan alone would not be helpful; however, the
addition of a sulfur colloid marrow scan or indium In 111 scan may be more specific to rule out
infection, though it is not warranted here. Total contact casting with non-weight-bearing or limited
weight bearing during Eichenholtz stage 1 when the foot is warm, erythematous, and swollen is
advised to help prevent deformity. Alternatively, stabilization with pneumatic bracing may also be
considered. While some authors have proposed early fixation or arthrodesis for Eichenholtz stage 1,
the gold standard is still total contact casting with no to limited weight bearing until the swelling
resolves and evidence of consolidation is seen on radiographs.

REFERENCES: Trepman E, Nihal A, Pinzur MS: Current topics review: Charcot neuropathy of the
foot and ankle. Foot Ankle Int 2005;26:46-63.
Richardson EG (ed): Orthopaedic Knowledge Update: Foot and Ankle 3. Rosemont, IL, American
Academy of Orthopaedic Surgeons, 2003, pp 123-134.
Simon SR, Tejwani SG, Wilson DL, et al: Arthrodesis as an early alternative to nonoperative
management of Charcot arthropathy of the diabetic foot. J Bone Joint Surg Am 2000;82:939-950.

29. Which of the following conditions is not associated with an increased risk of developing
Achilles tendinopathy?

1- Fluoroquinolone antibiotics
2- Diabetes mellitus
3- Obesity
4- Steroid exposure
5- Estrogen deficiency

PREFERRED RESPONSE: 5

DISCUSSION: Diabetes mellitus, obesity, and exposure to steroids have all been associated with the
development of Achilles tendinopathy. In addition, Achilles tendinopathy has been associated with a
history of hormone replacement therapy and the use of oral contraceptives. Quinolone antibiotics
have also been linked to Achilles tendinopathy.

REFERENCES: Holmes GB, Lin J: Etiologic factors associated with symptomatic Achilles
tendinopathy. Foot Ankle Int 2006;27:952-959.
Holmes GB, Mann RA, Well L: Epidemiological factors associated with rupture of the Achilles
tendon. Contemp Orthop 1991;23:327-331.
30. Figures a through c show the radiographs of a 38-year-old man following a motorcycle
accident. The posterior portion of the talus extruded through a posterolateral wound. The
extruded talar body is visible in the wound along with some road debris. Management
should now consist of surgical irrigation, debridement, and

1- removal of the extruded talus and placement of an external fixator.


2- immediate tibiocalcaneal fusion.
3- reimplantation of the talus, external fixation, and/or open reduction and internal
fixation of the talar neck fracture.
4- reimplantation followed by primary tibiotalar arthrodesis.
5- Syme amputation.

PREFERRED RESPONSE: 3

DISCUSSION: The extruded talus should be placed in sterile bacitracin solution, irrigated
thoroughly, gently debrided, and immediately replanted in the OR. Open reduction and internal
fixation of the talar fracture may be attempted immediately depending on the soft-tissue envelope, or
delayed after soft-tissue stabilization with an external fixator. A retrospective study of 19 patients
with an extruded talus reported that 12 patients had no subsequent surgery after definitive fixation,
7 had subsequent procedures, and 2 patients developed infections that were treated successfully at an
average of 42-month follow-up. Successful outcome in this series was attributed to multiple
debridements, soft-tissue stabilization, and primary wound closure.

REFERENCES: Smith CS, Nork SE, Sangeorzan BJ: The extruded talus: Results of reimplantation.
J Bone Joint Surg Am 2006;88:2418-2424.
Brewster NT, Maffulli N: Reimplantation of the totally extruded talus. J Orthop Trauma 1997;11:42-
45.
Marsh JL, Saltzman CL, Iverson M, et al: Major open injuries of the talus. J Orthop Trauma
1995;9:371-376.

31. Figures a and b show the radiographs of a patient who has pain with walking. On careful
questioning, it is determined that the discomfort occurs at push-off, or when the patient
attempts to climb stairs. What nonsurgical option is most likely to ameliorate
the symptoms?

1- Neutral posted rigid custom foot orthosis


2- Custom rigid UCBL foot orthosis
3- Ankle-foot orthosis with the ankle locked at 90 degrees
4- Shoe modification with a cushioned heel and rocker sole
5- Metal hinged/leather short ankle Arizona orthosis

PREFERRED RESPONSE: 4

DISCUSSION: The patient has a malunion of an attempted open reduction of a Lisfranc dislocation.
The pain occurs during the terminal stance phase of gait as load is being transferred from the
hindfoot to the forefoot. The bending moment can be best neutralized with shoe modification with a
cushioned heel and rocker sole, which best unloads the tarsal-metatarsal junction.

REFERENCES: Bono CM, Berberian WS: Orthotic devices: Degenerative disorders of the foot and
ankle. Foot Ankle Clin 2001;6:329-340.
Richardson EG (ed): Orthopaedic Knowledge Update: Foot and Ankle 3. Rosemont, IL, American
Academy of Orthopaedic Surgeons, 2004, pp 58-63.

32. An 18-year-old football player reports acute pain and swelling after a direct injury to his
plantar flexed foot. Examination reveals midfoot swelling and tenderness. Nonstanding
radiographs are normal. What is the next most appropriate step in management?

1- Gradual return to play


2- Physical therapy program
3- Custom orthosis
4- Non-weight-bearing cast
5- Weight-bearing AP radiograph
PREFERRED RESPONSE: 5

DISCUSSION: Differentiating between a midfoot sprain and Lisfranc diastasis is critical in the
management of the athlete with an acute injury to the midfoot. Greater than 2 mm of displacement
between the first and second metatarsals on a weight-bearing radiograph is an indication for
anatomic reduction with internal fixation of the tarsometatarsal joints. If no subluxation is noted,
treatment should consist of a non-weight-bearing cast for 6 weeks, followed by a gradual return
to activity.

REFERENCES: Mizel MS, Miller RA, Scioli MW (eds): Orthopaedic Knowledge Update: Foot and
Ankle 2. Rosemont, IL, American Academy of Orthopaedic Surgeons, 1998, pp 39-54.
Chiodo CP, Myerson MS: Developments and advances in the diagnosis and treatment of injuries
with the transmetatarsal joint. Orthop Clin North Am 2001;32:11-20.

33. A 36-year-old woman is wearing an ankle-foot orthosis for a foot drop secondary to spastic
hemiplegia following a postpartum stroke 2 years ago. Knee and hip motion and strength are
within normal ranges. She has undergone multiple rounds of physical therapy but has seen
no improvement over the past several months. No improvement has been recorded by
electromyography (EMG) studies over the past year. Examination reveals a 5-degree plantar
flexion contracture with clonus, heel varus, and compensatory knee hyperextension when
standing. She has 4/5 power in the tibialis anterior and gastrocnemius soleus complex with
resistance testing. Everters are 2/5 to resistance testing. EMG gait studies show that the
tibialis anterior demonstrates activity during both swing and stance phase that is increased
during swing phase. Premature firing of the triceps surae is noted when positioning the foot
in equinus prior to floor contact. What is the most appropriate management?

1- Percutaneous Achilles tendon lengthening


2- Percutaneous Achilles tendon lengthening and split tibialis anterior transfer to the
lateral cuneiform
3- Percutaneous Achilles tendon lengthening and interosseous posterior tibialis tendon
transfer to the peroneus tertius
4- Percutaneous Achilles tendon lengthening and tenotomy of the long toe flexor
tendons
5- Percutaneous Achilles tendon lengthening, tenotomy of the long toe flexors, and
Bridle procedure

PREFERRED RESPONSE: 2

DISCUSSION: The patient has a dynamic varus deformity secondary to spasticity of the tibialis
anterior during stance phase with inverter/everter imbalance. The patient still has active motion of
the tibialis anterior; therefore, an out-of-phase posterior tibial tendon transfer should not be
performed. The same is true of the Bridle procedure. Transfer of the posterior tibialis in this patient
may also result in subsequent planovalgus deformity. Lengthening of the Achilles tendon through a
percutaneous tenotomy will restore dorsiflexion and decrease clonus from the stretch response. If
adequate dorsiflexion is not obtained intraoperatively, then posterior tibialis tendon lengthening may
be considered. A split tibialis anterior tendon transfer to the lateral cuneiform, or, transfer of the
entire tendon to the cuneiform should correct the varus component and compensate for the weakened
peroneals.

REFERENCES: Yamamoto H, Okumura S, Morita S, et al: Surgical correction of foot deformities


after stroke. Clin Orthop Relat Res 1992;282:213-218.
Piazza SJ, Adamson RL, Moran MF, et al: Effects on tensioning errors in split transfers of tibialis
anterior and posterior tendons. J Bone Joint Surg Am 2003;85:858-865.
Morita S, Muneta T, Yamamoto H, et al: Tendon transfers for equinovarus deformed foot caused by
cerebrovascular disease. Clin Orthop Relat Res 1998;350:166-173.
34. A 52-year-old woman slipped on ice
in her driveway. Radiographs are
shown in Figures a and b. The patient
was treated in a short leg cast with
weight bearing as tolerated for 6
weeks. Due to persistent tenderness
at the fracture site, a CAM walker
was used for an additional 8 weeks.
Nine months after the injury, the
patient still walks with a limp and
reports pain with deep palpation at the
fracture site. What is the next most
appropriate step in management?

1- CT scan
2- Repeat period of immobilization
3- Referral to pain management for sympathetic blocks
4- Continued observation and physical therapy
5- Acupuncture

PREFERRED RESPONSE: 1

DISCUSSION: Persistent pain at the fracture site in the absence of infection is most likely due to a
nonunion, best detected by CT. Walsh and DiGiovanni reported on a series of closed rotational
fibular fractures in which nonunions were detected by CT in the absence of standard ankle
radiographic findings. Repeat immobilization would not be appropriate at this late date. Pain
management/sympathetic blocks would be considered if the patient displayed pain with light touch
and disproportionate pain consistent with a complex mediated pain syndrome. Acupuncture would
be expected to be of limited benefit.

REFERENCES: Walsh EF, DiGiovanni C: Fibular nonunion after closed rotational ankle fracture.
Foot Ankle Int 2004;25:488-495.
Richardson EG (ed): Orthopaedic Knowledge Update: Foot and Ankle 3. Rosemont, IL, American
Academy of Orthopaedic Surgeons, 2003, pp 182-183.

35. What is the most frequent complication of percutaneous repair of an acute Achilles tendon
rupture?

1- Sural nerve entrapment


2- Re-rupture
3- Infection
4- Suture granuloma
5- Wound healing complications
PREFERRED RESPONSE: 1

DISCUSSION: Sural nerve entrapment is the major risk of percutaneous repair. A small mini-open
technique with a suture guide can obviate that issue. Re-rupture rates after surgical repair are
approximately 3%. Infection and wound problems are rarely encountered with percutaneous repair;
they are issues with open repair.

REFERENCES: Aracil J, Pina A, Lozano JA, et al: Percutaneous suture of Achilles tendon ruptures.
Foot Ankle 1992;13:350-351.
Sutherland A, Maffulli N: A modified technique of percutaneous repair of the ruptured Achilles
tendon. Oper Orthop Traumatol 1998;10:50-58.
Assal M, Jung M, Stern R, et al: Limited open repair of Achilles tendon ruptures: A technique with a
new instrument and findings of a prospective multicenter study. J Bone Joint Surg Am
2002;84:161-170.

36. A 2-year-old child is brought in by his parents for evaluation of intoeing. The child has a
normal neuromuscular examination, but the heel bisector line is in the fourth web space,
indicating a severe flexible metatarsus adductus deformity. The remainder of the lower
extremity examination is unremarkable. What is the most appropriate treatment?

1- Observation as the deformity should resolve in time


2- Wearing of straight last shoes
3- Serial stretching and casting for the next 6 to 12 weeks
4- Heyman, Herndon, and Strong capsular release at the tarsometatarsal and
intermetatarsal joints
5- Berman and Gartland dome-shaped osteotomies of the metatarsal bases

PREFERRED RESPONSE: 3

DISCUSSION: Weinstein reported on 31 patients (45 feet) with congenital metatarsus adductus
followed for an average of 33 years. Twenty-nine feet had moderate to severe deformities treated
with manipulation and casting with a 90% success rate. In a young child, surgery is not indicated
until nonsurgical management has failed. In patients 2 to 4 years of age, tarsometatarsal
capsulotomies are indicated, whereas multiple metatarsal osteotomies are reserved for recalcitrant
deformities in children older than 4 years of age. Mild or moderate metatarsus adductus that is
passively correctable will resolve without treatment.

REFERENCES: Beaty J: Congenital anomalies of the lower extremity, in Canale ST (ed):


Campbells Operative Orthopaedics, ed 10. Philadelphia PA, Mosby, 2003, pp 983-988.
Katz K, David R, Soudry M: Below-knee plaster cast for the treatment of metatarsus adductus.
J Pediatr Orthop 1999;19:49-50.
Weinstein SL: Bristol-Myers Squibb/Zimmer award for distinguished achievement in orthopaedic
research. Long-term follow-up of pediatric orthopaedic conditions: Natural history and outcomes
of treatment. J Bone Joint Surg Am 2000;82:980-990.
37. A 34-year-old man has had a 13-month history of an equinovarus deformity of the foot and
ankle after a motorcycle accident. His foot and ankle are flexible, but bracing has become
uncomfortable. Active dorsiflexion and eversion are absent. What is the most appropriate
treatment?

1- Ankle arthrodesis
2- Subtalar arthrodesis
3- Pantalar arthrodesis
4- Posterior tibialis tendon transfer to the lateral midfoot with Achilles tendon
lengthening
5- Split anterior tibialis tendon transfer to the lateral midfoot with Achilles tendon
lengthening

PREFERRED RESPONSE: 4

DISCUSSION: Arthrodesis of any of the ankle or hindfoot joints should be reserved for fixed
deformities or end-stage degenerative arthritis. Achilles tendon lengthening is necessary to correct
the equinus and to improve dorsiflexion-plantar flexion balance. Similarly, transfer of the posterior
tibialis tendon reduces both plantar flexion and inversion torque.

REFERENCES: Hansen ST: Function Reconstruction of the Foot and Ankle. Philadelphia, PA,
Lippincott Williams & Wilkins, 2000, pp 442-447.
Atesalp AS, Yildiz C, Komurcu M, et al: Posterior tibial tendon transfer and tendo-Achilles
lengthening for equinovarus foot deformity due to severe crush injury. Foot Ankle Int
2002;23:1103-1106.

38. Figures a and b show the radiographs of a 14-year-old boy who sustained a twisting injury to
his ankle. If attempted closed reduction is unsuccessful, what is the primary reason to
proceed with surgical treatment?

1- Decreased risk of posttraumatic arthritis


2- Avoid growth arrest
3- Minimize risk of osteonecrosis
4- Prevent clinical internal rotation deformity
5- Prevent development of a late tarsal tunnel syndrome

PREFERRED RESPONSE: 1

DISCUSSION: Triplane fractures generally occur in children who are near skeletal maturity. The
injury is generally caused by a supination external rotation mechanism. The number of fracture
fragments present (two or three) depends on what part of the physes is closed at the time of injury.
Articular congruity is the major concern in the management of these injuries since the patient has
almost reached skeletal maturity. The goal is to restore articular congruity to minimize the
development of posttraumatic arthritis.

REFERENCES: Vaccaro A (ed): Orthopaedic Knowledge Update 8. Rosemont, IL, American


Academy of Orthopaedic Surgeons, 2005, pp 757-765.
Kling TF Jr, Bright RW, Hensinger RN: Distal tibial physeal fractures in children that may require
open reduction. J Bone Joint Surg Am 1984;66:647-657.
Spiegel PG, Mast JW, Cooperman DR, et al: Triplane fractures of the distal tibial epiphysis.
Clin Orthop Relat Res 1984;188:74-89.
39. A 75-year-old woman reports foot pain and states that her foot has become progressively
flatter in the past 3 years. Custom inserts and physical therapy have failed to provide
relief. Examination reveals a flexible hindfoot and mild heel cord contracture. The patient is
able to perform a single limb heel rise. Weight-bearing radiographs are shown in Figures a
through d. What is the most appropriate surgical management?

1- Posterior tibial tendon debridement and synovectomy


2- Tendon transfer, spring ligament repair, and heel cord lengthening
3- Tendon transfer, lateral column lengthening, and heel cord lengthening
4- Realignment triple arthrodesis and heel cord lengthening
5- Medial column arthrodesis and heel cord lengthening

PREFERRED RESPONSE: 5

DISCUSSION: The patient has end-stage midfoot arthritis, with a secondary flatfoot deformity
through the midfoot. The ability to perform a single limb heel rise indicates that the posterior tibial
tendon is functioning, and the weight-bearing radiographs show normal calcaneal pitch and talar
head coverage, thus confirming that the flatfoot deformity is isolated to the midfoot. Therefore, the
most appropriate treatment is medial column arthrodesis and heel cord lengthening. The other listed
procedures are not indicated because they are used in the management of adult flatfoot from
posterior tibial tendon insufficiency.

REFERENCES: Toolan BC: Midfoot arthrodesis: Challenges and treatment alternatives. Foot Ankle
Clin 2002;7:75-93.
Horton GA, Olney BW: Deformity correction and arthrodesis of the midfoot with a medial plate.
Foot Ankle 1993;14:493-499.

40. A 52-year-old woman who underwent cheilectomy 1 year ago for hallux rigidus now reports
continued pain in the first metatarsophalangeal joint. She did not have any incision healing
problems, and has not had any fevers, erythema, or drainage. Which of the following
procedures will provide the best combination of pain relief and function?
1- First metatarsophalangeal arthrodesis
2- Soft-tissue interposition arthroplasty
3- First metatarsophalangeal total joint arthroplasty
4- First metatarsophalangeal resurfacing hemiarthroplasty
5- Proximal phalanx dorsiflexion osteotomy (Moberg)

PREFERRED RESPONSE: 1

DISCUSSION: All but the Moberg osteotomy are capable of providing pain relief; however,
arthrodesis offers the best long-term results and restores weight bearing and propulsion function to
the first ray.

REFERENCES: Machacek F Jr, Easley ME, Gruber F, et al: Salvage of a failed Keller resection
arthroplasty. J Bone Joint Surg Am 2004;86:1131-1138.
Myerson MS, Schon LC, McGuigan FX, et al: Result of arthrodesis of the hallux
metatarsophalangeal joint using bone graft for restoration of length. Foot Ankle Int
2000;21:297-306.
41. During a posterior approach to the right Achilles
tendon, the surgeon encounters a nerve running
with the small saphenous vein as shown in
Figure. This nerve innervates what part of the
foot?

1- Posterior heel
2- Plantar-lateral foot
3- Plantar-medial foot
4- Dorso-lateral foot
5- Dorso-medial foot

PREFERRED RESPONSE: 4

DISCUSSION: The sural nerve runs with the small saphenous vein on the posterior leg just lateral to
the Achilles tendon. It is formed by contributions from both the tibial and common peroneal nerves
and provides sensation on the dorso-lateral aspect of the foot.

REFERENCES: Aktan Ikiz ZA, Ucerler H, Bilge O: The anatomic features of the sural nerve with
an emphasis on its clinical importance. Foot Ankle Int 2005;26:560-567.
Lawrence SJ, Botte MJ: The sural nerve in the foot and ankle: An anatomic study with clinical and
surgical implications. Foot Ankle Int 1994;15:490-494.

42. A 23-year-old woman has had a 14-month history of ankle pain after surgical treatment of
multiple injuries resulting from a motor vehicle accident. Weight bearing began 4 months
after surgery. The pain occurs with weight bearing and motion, but there is very little pain at
rest. She has no pertinent medical history and does not smoke. Figures a and b show current
radiographs. What is the most appropriate surgical option?
1- Talectomy
2- Revision open reduction and internal fixation (ORIF) with bone grafting
3- Ankle arthrodesis
4- Tibiotalocalcaneal arthrodesis
5- Triple arthrodesis

PREFERRED RESPONSE: 2

DISCUSSION: The radiographs reveal nonunion of a talar neck fracture. There is no radiographic
evidence of osteonecrosis or significant degenerative arthritis. The results of talectomy are
suboptimal. Arthrodesis would be indicated for degenerative arthritis. Revision ORIF is feasible
and preserves motion. A vascularized graft should be considered whenever osteonecrosis is present,
but the talar body appears viable in this case.

REFERENCES: Calvert E, Younger A, Penner M: Post talus neck fracture reconstruction.


Foot Ankle Clin 2007;12:137-151.
Migues A, Solari G, Carrasco NM, et al: Repair of talar neck nonunion with indirect
corticocancellous graft technique: A case report and review of the literature. Foot Ankle Int
1996;17:690-694.

43. What type of physical therapy is most effective for chronic noninsertional Achilles
tendinopathy?

1- Stair climbing
2- Eccentric strengthening
3- Concentric strengthening
4- Isometric strengthening
5- Rope jumping

PREFERRED RESPONSE: 2

DISCUSSION: Eccentric gastrocsoleus strengthening (especially with heavy loads) consistently has
been shown to be superior in the management of Achilles tendinopathy. Decreases in pain and
increases in strength have been demonstrated despite the frequently refractory nature of this
condition.

REFERENCES: Richardson EG (ed): Orthopaedic Knowledge Update: Foot and Ankle 3.


Rosemont, IL, American Academy of Orthopaedic Surgeons, 2003, pp 91-102.
Vora AM, Myerson MS, Oliva F, et al: Tendinopathy of the main body of the Achilles tendon.
Foot Ankle Clin 2005;10:293-308.
44. A 27-year-old man was struck by a taxi cab and sustained comminuted right distal third tibia
and fibula fractures; treatment consisted of placement of an intramedullary nail in the tibia
the following morning. At his 6-month follow-up, he has clawing of all five toes.
Examination reveals flexion deformities of the distal and proximal interphalangeal joints that
are flexible with plantar flexion and rigid with dorsiflexion. Calluses are present on the
dorsum and tip of the toes. Single heel rise is normal. He has a mild equinus contracture
(relative to the left leg) that is not relieved with knee flexion. What is the most appropriate
treatment option?

1- Physical therapy and bracing


2- Reassurance that the deformity will resolve with time
3- Achilles tendon lengthening, and release or retromalleolar lengthening of the flexor
digitorum longus (FDL) and flexor hallucis longus (FHL)
4- FDL and FHL tenotomies at the individual digits with transfer of the posterior tibial
tendon to the dorsum of the foot
5- FDL and FHL tenotomies at the individual digits with midfoot capsular release and
hallux interphalangeal fusion

PREFERRED RESPONSE: 3

DISCUSSION: This is an example of tethering of the flexor hallucis longus/flexor digitorum longus
(FHL/FDL) to the fracture site. Additional time and/or physical therapy and bracing would not be
expected to be of benefit. Release of the FHL and FDL from the fracture site or retromalleolar
lengthening will address the posttraumatic claw toe deformity and Achilles tendon lengthening will
address the mild equinus. Posterior tibial tendon transfer is not appropriate as the patient
demonstrates a normal heel rise. Midfoot releases and hallux fusion are also not indicated.

REFERENCES: Feeny MS, Williams RL, Stephens MM: Selective lengthening of the proximal
flexor tendon in the management of acquired claw toes. J Bone Joint Surg Br 2001;83:335-338.
Clawson DK: Claw toes following tibial fracture. Clin Orthop Relat Res 1974;103:47-48.

45. A 24-year-old man reports the development of a foot drop following a knee dislocation
1 year ago. The common peroneal nerve was found to be in continuity at the time of surgical
reconstruction of the posterolateral corner of the knee joint. He would like to eliminate the
need for an ankle-foot orthosis. What is the best option to achieve elimination of the
orthosis?

1- Repeat neurolysis of the common peroneal nerve at the knee level


2- Repeat neurolysis of the common peroneal nerve with cable grafting
3- Extensor hallucis longus transfer to the distal first metatarsal
4- Anterior transfer of the tibialis posterior tendon through the interosseous membrane
5- Ankle fusion
PREFERRED RESPONSE: 4

DISCUSSION: The ankle dorsiflexor muscles have been denervated for too long a period to expect
reinnervation to be successful. Even if the extensor hallucis longus tendon was functional, it is
unlikely to have sufficient strength to achieve dynamic ankle dorsiflexion. The tibialis posterior
tendon transfer has been shown to predictably achieve these goals in a high percentage of patients.
Successful ankle fusion is likely to fail with time due to the development of forefoot equinus.

REFERENCES: Pinzur MS, Kett N, Trilla M: Combined anteroposterior tibial tendon transfer in
post-traumatic peroneal palsy. Foot Ankle 1988;8:27l-275.
Lipscomb P, Sanchez J: Anterior transplantation of the posterior tibial tendon for persistant palsy of
the common peroneal nerve. J Bone Joint Surg Am 1961;43:60-66.
46. A 21-year-old male construction worker fell from a roof and sustained an injury to his left
foot. Radiographs and CT scans are shown in Figures a through e. Compared to nonsurgical
management, surgical treatment offers which of the following advantages?

1- Quicker return to activities


2- Quicker return to work
3- Increased subtalar joint range of motion
4- Decreased risk of nonunion
5- Decreased risk of posttraumatic arthritis

PREFERRED RESPONSE: 5

DISCUSSION: The radiographs and CT scans show a displaced intra-articular calcaneal fracture,
with loss of calcaneal height and length. Recent multicenter, randomized, prospective studies
suggest that surgical treatment of displaced intra-articular calcaneal fractures is associated with an
almost six-fold decrease in the risk of posttraumatic subtalar arthritis (necessitating subtalar
arthrodesis) compared to nonsurgical treatment. Despite ongoing controversy, surgical treatment has
not been shown to be advantageous with respect to activity, time to return to work, or subtalar joint
range of motion. A nonunion of a calcaneal fracture is exceedingly rare regardless of the treatment
method.

REFERENCES: Buckley R, Tough S, McCormack R, et al: Operative compared with nonoperative


treatment of displaced intra-articular calcaneal fractures: A prospective, randomized, controlled
multicenter trial. J Bone Joint Surg Am 2002;84:1733-1744.
Csizy M, Buckley R, Tough S, et al: Displaced intra-articular calcaneal fractures: Variables
predicting late subtalar fusion. J Orthop Trauma 2003;17:106-112.

47. A 51-year-old plumber has a failed peroneus brevis tendon repair. He reports continued pain
and swelling in the distal retrofibular area. MRI shows longitudinal tears of the peroneus
longus and peroneus brevis. What is the surgical treatment of choice at this time?

1- Subtalar fusion
2- Posterior tibial tendon transfer to the cuboid
3- Split posterior tibial tendon transfer to the lateral cuneiform
4- Flexor digitorum longus transfer to the fifth metatarsal
5- Excision of both the peroneus longus and brevis

PREFERRED RESPONSE: 4

DISCUSSION: A flexor digitorum longus transfer, while not as strong as the peroneals, improves
the tendon balance and maintains hindfoot mobility. Subtalar fusion is a salvage procedure.
Posterior tibial tendon transfer compromises inversion strength and arch height. Functional absence
of the peroneals results in an imbalance that could lead to forefoot varus.

REFERENCES: Redfern D, Myerson M: The management of concomitant tears of the peroneus


longus and brevis tendons. Foot Ankle Int 2004;25:695-707.
Borton DC, Lucas P, Jomha NM, et al: Operative reconstruction after transverse rupture of the
tendons of both peroneus longus and brevis: Surgical reconstruction by transfer of the flexor
digitorum longus tendon. J Bone Joint Surg Br 1998;80:781-784.

48. Which of the following imaging modalities is most accurate in locating a toothpick in the
plantar arch of the foot?

1- Radiograph
2- CT
3- MRI
4- Ultrasound
5- Bone scan

PREFERRED RESPONSE: 4

DISCUSSION: Ultrasound is best at imaging abrupt changes in the density of adjacent tissue and
therefore is best at imaging wood in the soft tissues of the foot.

REFERENCES: Mizel MS, Steinmetz ND, Trepman E: Detection of wooden foreign bodies in
muscle tissue: Experimental comparison of computed tomography, magnetic resonance imaging, and
ultrasonography. Foot Ankle Int 1994;15:437-443.
Jacobson JA, Powell A, Craig JG, et al: Wooden foreign bodies in soft tissue: Detection at US.
Radiology 1998;206:45-48.
49. A 35-year-old man is seen in the emergency department with a bullet wound to the foot that
occurred 2 hours ago. Examination reveals a 0.5-cm entrance wound on the dorsum of the
foot and a 1.5-cm exit wound on the plantar aspect. Exploration of the plantar wound in the
emergency department reveals bone and metal fragments. Radiographs reveal a comminuted,
unstable fracture of the base of the first metatarsal and cuneiform. Management should
consist of tetanus toxoid, and

1- surface irrigation, sterile dressing, and a short leg cast.


2- surface irrigation, sterile dressing, a short leg cast, and oral antibiotics.
3- surface irrigation, sterile dressing, a short leg cast, and IV antibiotics.
4- surgical debridement, a short leg cast, and IV antibiotics.
5- surgical debridement, external or internal fixation, and IV antibiotics.

PREFERRED RESPONSE: 5

DISCUSSION: The patient sustained a type I unstable fracture that requires debridement of
superficial fragments from the sole and surgical stabilization. Low-velocity wounds less than 8
hours old are considered type I open fractures. In contrast, gunshot wounds with associated fractures
more than 8 hours old are considered type II open fractures using the Gustilo and Anderson
classification. Gustilo type I stable fractures due to gunshot wounds and seen within 8 hours can be
treated with tetanus toxoid (if no history of immunization or booster within 5 years), surface
irrigation, and casting or a hard sole shoe. Antibiotics are not required unless gross contamination is
present. However, if the extent of contamination is unclear, or if a joint is penetrated, then routine
antibiotic prophylaxis is recommended. Indications for surgery include: articular involvement,
unstable fractures, presentation 8 or more hours after injury, tendon involvement, and superficial
fragments in the palm or sole. Type I unstable fractures may be stabilized with internal or external
fixation. Type II unstable fractures should be treated with external fixation and repeat debridements
until clean.

REFERENCES: Holmes GB Jr: Gunshot wounds of the foot. Clin Orthop Relat Res
2003;408:86-91.
Bartlett CS, Helfet DL, Hausman MR, et al: Ballistics and gunshot wounds: Effects on
musculoskeletal tissues. J Am Acad Orthop Surg 2000;8:21-36.

50. What is the most frequent location of entrapment of the deep peroneal nerve?

1- Tendon of the extensor hallucis brevis


2- Inferior extensor retinaculum
3- Osteophytes of the talonavicular joint
4- Os intermetatarseum
5- Base of the fifth metatarsal
PREFERRED RESPONSE: 2

DISCUSSION: The most frequently described entrapment of the deep peroneal nerve is the anterior
tarsal tunnel syndrome. This syndrome refers to entrapment of the deep peroneal nerve under the
inferior extensor retinaculum. Entrapment can also occur as the nerve passes under the tendon of the
extensor hallucis brevis. Compression by underlying dorsal osteophytes of the talonavicular joint
and an os intermetatarseum (between the bases of the first and second metatarsals) have previously
been described in runners.

REFERENCES: Kopell HP, Thompson WA: Peripheral entrapment neuropathies of the lower
extremity. N Engl J Med 1960;262:56-60.
Schon LC, Mann RA: Diseases of the nerves, in Coughlin MJ, Mann RA, Saltzman CL (eds):
Surgery of the Foot and Ankle, ed 8. Philadelphia, PA, Mosby-Elsevier, 2007, vol 1, pp 675-677.

51. What is the most common malignant tumor of the foot?

1- Chondrosarcoma
2- Synovial sarcoma
3- Osteosarcoma
4- Clear cell sarcoma
5- Melanoma

PREFERRED RESPONSE: 5

DISCUSSION: Whereas chondrosarcoma is the most frequently occurring malignant bone tumor of
the foot and synovial sarcoma is the most common soft-tissue foot malignancy, the most common
malignant tumor overall is melanoma. It constitutes approximately 25% of lesions found on the
lower extremity. Furthermore, 31% of all melanomas arise in the foot.

REFERENCES: Mizel MS, Miller RA, Scioli MW (eds): Orthopaedic Knowledge Update: Foot and
Ankle 2. Rosemont, IL, American Academy of Orthopaedic Surgeons, 1998, pp 11-26.
Bos GD, Ester RJ, Woll TS: Foot tumors: Diagnosis and treatment. J Am Acad Orthop Surg
2002;10:259-270.

52. A 40-year-old man underwent an ankle arthroscopy 6 months ago for a talus osteochondral
defect. He continues to have pain and burning on the lateral portal but states that the pain is
now more superficial than his original pain. Examination reveals that he has shooting pain to
his medial foot and ankle when his lateral portal is tapped. A previous injection around the
lateral portal gave him relief for about 2 weeks. What treatment will best eliminate his pain?

1- Neuroplasty of the sural nerve


2- Neuroplasty or excision and burial of the deep peroneal nerve
3- Neuroplasty or excision and burial of the medial branch of the superficial peroneal
nerve
4- Repeat arthroscopy for worsening of the talus osteochondral defect
5- Anaesthetic skin patches

PREFERRED RESPONSE: 3

DISCUSSION: The patient clearly has entrapment of the superficial peroneal nerve in the lateral
portal. It is most likely only the medial branch by examination. If the nerve is in good condition, it
can simply be released. If the nerve is cut or severely thinned, it is better excised and buried. The
sural nerve most likely would be caught in a posterior-lateral portal.

REFERENCES: Jobe MT, Wright PE: Peripheral nerve injuries, in Canale ST (ed): Campbells
Operative Orthopaedics. St Louis, MO, Mosby, 1998, pp 3839-3844.
Saito A, Kikuchi S: Anatomic relations between ankle arthroscopic portal sites and the superficial
peroneal and saphenous nerves. Foot Ankle Int 1998;19:748-752.

53. When performing a Weil osteotomy of a lesser metatarsal, the desired angle of the saw cut
should be approximately

1- perpendicular to the shaft of the metatarsal.


2- parallel with the inclination of the metatarsal.
3- parallel with the plantar surface of the foot.
4- 45 degrees to the shaft of the metatarsal.
5- 10 degrees to the shaft of the metatarsal.

PREFERRED RESPONSE: 3

DISCUSSION: Appropriate orientation of the saw cut when performing a Weil osteotomy is
approximately parallel with the plantar surface of the foot. This is done in an effort to minimize
plantar displacement of the capital fragment. The removal of additional bone from the osteotomy
site either by removing a separate wafer of bone or using a thicker saw blade has also been described
to minimize plantar displacement of the distal fragment.

REFERENCES: Trnka H, Nyska M, Parks BG, et al: Dorsiflexion contracture after the Weil
osteotomy: Results of cadaver study and three-dimensional analysis. Foot Ankle Int 2001;22:47-50.
Grimes J, Coughlin M: Geometric analysis of the Weil osteotomy. Foot Ankle Int 2006;27:985-992.

54. A patient with diabetic peripheral neuropathy undergoes a partial first ray amputation for a
chronic ulcer beneath the first metatarsal head. The insertion of the anterior tibialis is
preserved. The patient has 10 degrees of passive dorsiflexion at the ankle and no other
foot deformities or ulcers. Which of the following is considered appropriate shoe wear for
this patient?

1- Snug fitting shoe to block side-to-side motion that is common after this procedure
2- Leather sole shoe to prevent catching the shoe on carpet which is common with
crepe-soled shoes
3- Lateral wedge on the shoe to offset the external rotation during the toe-off phase of
gait commonly seen after this procedure
4- Steel shank to extend the foot lever and prevent deformity at the toe break
5- Custom-made shoe to provide the best possible fit and function

PREFERRED RESPONSE: 4

DISCUSSION: The steel shank is a flat 1-inch steel strip placed between layers of the shoe to extend
the foot lever and prevent deformity at the toe break seen following a partial first ray amputation. A
rocker sole may be added as well to facilitate transition from foot flat to the toe-off phase of gait.
Proper shoe fit is important, but snug fitting shoes in a patient with peripheral neuropathy and
likely fluctuations in volume from intermittent swelling are to be avoided. A custom shoe is an
unnecessary expense. The patient has at least 10 degrees of dorsiflexion at the ankle with an intact
anterior tibialis muscle; therefore, catching the sole on carpeting should not be a problem.

REFERENCES: Philbin TM, Leyes M, Sferra JJ, et al: Orthotic and prosthetic devices in partial foot
amputations. Foot Ankle Clin 2001;6:215-228.
Pinzur MS, Dart HC: Pedorthic management of the diabetic foot. Foot Ankle Clin 2001;6:205-214.

55. A 32-year-old laborer reports left ankle pain and deformity. History reveals that he
sustained a left ankle fracture 2 years ago and was treated with closed reduction and
casting. Radiographs are shown in Figures a through c. What is the most
appropriate management?

1- Bracing and physical therapy


2- Intra-articular injection of steroids into the ankle joint, bracing, and physical therapy
3- Intra-articular injection of hyaluronic acid product into the ankle joint, bracing, and
physical therapy
4- Ankle fusion
5- Corrective osteotomy of the fibula and medial malleolus with reconstruction of the
syndesmosis if unstable

PREFERRED RESPONSE: 5

DISCUSSION: Corrective osteotomy of fibular malunions, with appropriate lengthening, even in the
presence of early arthritis, has been shown to decrease ankle pain and increase stability. Reduction
and bone grafting of the medial malleolar nonunion is also needed. There is no evidence supporting
the use of intra-articular steroids or hyaluronic acid in the ankle joint. Lateral talar displacement of
even 1 mm has been reproducibly shown to decrease tibiotalar contact by 40% to 42%, causing a
predisposition to arthritis.

REFERENCES: Weber D, Friederich NF, Muller W: Lengthening osteotomy of the fibula for post-
traumatic malunion: Indication, technique and results. Int Orthop 1998;22:149-152.
Lloyd J, Elsayed S, Hariharan K, et al: Revisiting the concept of talar shift in ankle fractures. Foot
Ankle Int 2006;27:793-796.
Offierski CM, Graham JD, Hall JH, et al: Later revision of fibular malunion in ankle fractures. Clin
Orthop Relat Res 1982;171:145-149.
Yablon IG, Leach RE: Reconstruction of malunited fractures of the lateral malleolus. J Bone Joint
Surg Am 1989;71:521-527.

56. Preservation or reconstruction of which of the following structures is essential to minimize


the risk of hallux valgus developing after removal of part or all of the medial sesamoid?

1- Flexor hallucis longus tendon


2- Flexor hallucis brevis tendon
3- Abductor hallucis tendon
4- Adductor hallucis tendon
5- Extensor hallucis brevis tendon

PREFERRED RESPONSE: 2

DISCUSSION: Complications of medial sesamoidectomy include stiffness, claw toe, and hallux
valgus. Each sesamoid sits within its respective head of the flexor hallucis brevis tendon. Excision
of one sesamoid can result in slack in its flexor hallucis brevis tendon; therefore, it is imperative to
preserve or repair the flexor hallucis brevis tendon when removing the medial sesamoid.

REFERENCES: Dedmond BT, Cory JW, McBryde A Jr: The hallucal sesamoid complex. J Am
Acad Orthop Surg 2006;14:745-753.
Lee S, James WC, Cohen BE, et al: Evaluation of hallux alignment and functional outcome after
isolated tibial sesamoidectomy. Foot Ankle Int 2005;26:803-809.
57. In the nonsurgical management of posterior tibial tendon dysfunction with flexible deformity,
a common strategy is to prescribe an ankle-foot orthosis or a University of California
Biomechanics Laboratory (UCBL) orthosis with medial posting. A high patient satisfaction
rating and favorable outcome with this nonsurgical management is most likely in which of
the following situations?

1- Relatively young, active patient


2- Patient with an inflammatory systemic disorder
3- Elderly patient with a sedentary lifestyle
4- Patient with severe arthritis of the ipsilateral hip or knee
5- Patient with Parkinsons disease

PREFERRED RESPONSE: 3

DISCUSSION: Most authors recommend an initial trial of nonsurgical management in the treatment
of adult-acquired flatfoot deformity such as posterior tibial tendon dysfunction. Chao and associates
found that there is high patient satisfaction with ankle-foot orthoses and UCBL-type inserts in
elderly patients with a relatively sedentary lifestyle. Alternatively, there was a higher dissatisfaction
rate in young active patients, those with balance and ambulation difficulties (Parkinsons, severe
arthritis of the hip or knee), and patients with inflammatory systemic disorders.

REFERENCES: Chao W, Wapner KL, Lee TH, et al: Nonoperative management of posterior tibial
tendon dysfunction. Foot Ankle Int 1996;17:736-741.
Noll KH: The use of orthotic devices in adult acquired flatfoot deformity. Foot Ankle Clin
2001;6:25-36.

58. Figure shows the clinical photograph of a patient who has


developed a residual limb ulcer following a traumatic
transtibial amputation 2 years ago. What is the preferred
treatment to resolve the ulcer?

1- Avoid wearing the prosthesis until the ulcer is healed


and perform local wound care.
2- Obtain a new prosthesis with an energy-storing foot
to dampen impact.
3- Perform local wound care in conjunction with
modification of the prosthetic socket and cushioned
liner.
4- Excise the wound and advance the soft-tissue
envelope.
5- Perform a distal tibiofibular bone bridge and advance
the soft-tissue envelope.
PREFERRED RESPONSE: 3

DISCUSSION: The first step in the treatment of an amputation residual limb (stump) ulcer is local
wound care and adjustment of the residual limb-prosthetic interface, as well as adjusting prosthetic
alignment. Surgical revision should be undertaken only when prosthetic modification is
unsuccessful.

REFERENCES: Murnaghan JJ, Bowker JH: Musculoskeletal complications, in Smith DG, Michael
JW, Bowker JH (eds): Atlas of Amputations and Limb Deficiencies, ed 3. Rosemont, IL, American
Academy of Orthopaedic Surgeons, 2004, pp 683-700.
Smith DG, Ferguson JR: Transtibial amputations. Clin Orthop Relat Res 1999;361:108-115.
59. The spring ligament of the foot connects what two bones?

1- Tibia and talus


2- Talus and navicular
3- Talus and calcaneus
4- Calcaneus and cuboid
5- Calcaneus and navicular

PREFERRED RESPONSE: 5

DISCUSSION: The spring ligament is also known as the calcaneonavicular ligament and connects
the calcaneus to the navicular. This ligament supports the talar head and is an important anatomic
supporting structure of the medial longitudinal arch of the foot.

REFERENCES: Choi K, Lee S, Otis JC, et al: Anatomical reconstruction of the spring ligament
using peroneus longus tendon graft. Foot Ankle Int 2003;24:430-436.
Davis WH, Sobel M, DiCarlo EF, et al: Gross, histological and microvascular anatomy and
biomechanical testing of the spring ligament complex. Foot Ankle Int 1996;17:95-102.

60. An obese 62-year-old man reports a 10-year history of progressive flatfoot deformity and a
3-month history of a painful callus along the plantar medial midfoot that has not improved
with custom shoe wear, pedorthics, and callus care. There is no hindfoot motion, but
functional ankle motion remains. He does not have diabetes mellitus. Radiographs are
shown in Figures a and b. What is the best surgical option at this point?

1- Exostectomy
2- Lateral column lengthening
3- Medial slide calcaneal osteotomy
4- Talonavicular arthrodesis
5- Triple arthrodesis

PREFERRED RESPONSE: 5

DISCUSSION: The deformity is long-standing, the hindfoot is immobile, and the radiographs reveal
severe degenerative arthritis involving the entire hindfoot, severe deformity, and talonavicular
dislocation. The exostosis responsible for the callus is the talar head; resection would severely
destabilize the foot. Degenerative arthritis and fixed deformity preclude lateral column lengthening,
medial slide calcaneal osteotomy, and talonavicular arthrodesis. Triple arthrodesis is the only viable
option.

REFERENCES: Johnson JE, Yu JR: Arthrodesis techniques in the management of Stage II and III
acquired adult flatfoot deformity. Instr Course Lect 2006;55:531-542.
Pinney SJ, Lin SS: Current concept review: Acquired adult flatfoot deformity. Foot Ankle Int
2006;27:66-75.

61. A 20-year-old collegiate football player sustains an injury to his left foot 3 weeks before the
start of the fall season. Examination reveals localized tenderness over the lateral midfoot and
normal foot alignment. Radiographs are shown in Figures a through c. What is the treatment
of choice?

1- Intramedullary screw fixation


2- Onlay bone graft
3- Application of a walking boot with weight bearing as tolerated
4- Application of a short leg cast with weight bearing as tolerated
5- Application of a short leg cast and non-weight-bearing

PREFERRED RESPONSE: 1

DISCUSSION: Due to the relatively high incidence of delayed union and nonunion associated with
this mildly displaced Jones-type fracture, and the temporal proximity to his playing season,
intramedullary screw fixation is the treatment of choice in this collegiate athlete to best ensure
healing and expedite his return to football. If nonsurgical management were elected, application of a
non-weight-bearing short leg cast would be appropriate since a higher likelihood of healing is
expected with it versus a short leg walking cast. The risk of recurrent fracture of fractures that heal
with nonsurgical management has reportedly been high (approximately 30%).

REFERENCES: Quill GE: Fractures of the proximal fifth metatarsal. Orthop Clin North Am
1995;26:353-361.
Torg JS, Balduini FC, Zelko RR, et al: Fractures of the base of the fifth metatarsal distal to the
tuberosity: Classification and guidelines for nonsurgical and surgical management. J Bone Joint
Surg Am 1984;66:209-214.
Dameron TB Jr: Fractures of the proximal fifth metatarsal: Selecting the best treatment option.
J Am Acad Orthop Surg 1995;3:110-114.

62. When the great toe deviates into a valgus position, the action of the abductor hallucis muscle
becomes one of

1- increased abduction.
2- pronation.
3- flexion.
4- flexion and pronation.
5- extension.

PREFERRED RESPONSE: 4

DISCUSSION: The abductor hallucis muscle inserts together with the medial tendon of the flexor
hallucis brevis into the medial base of the proximal phalanx of the great toe. When the hallux
assumes a valgus position, the action of the abductor becomes one of flexion and pronation of the
first metatarsal.

REFERENCES: Resch S: Functional anatomy and topography of the foot and ankle, in Myerson M
(ed): Foot and Ankle Disorders. Philadelphia, PA, WB Saunders, 2000, vol 1, pp 25-49.
Sarrafian SK: Anatomy of the Foot and Ankle: Descriptive, Topographic, Functional, ed 2.
Philadelphia, PA, JB Lippincott, 1993.

63. When performing a bunionectomy with a release of the lateral soft-tissue structures, the
surgeon is cautioned against releasing the conjoined tendon that inserts along the lateral
base of the proximal phalanx of the great toe. This conjoined tendon is made up of what
two muscles?

1- Flexor hallucis longus and flexor hallucis brevis


2- Flexor hallucis longus and adductor hallucis
3- Flexor hallucis brevis and adductor hallucis
4- Flexor hallucis longus and abductor hallucis
5- Flexor hallucis brevis and abductor hallucis

PREFERRED RESPONSE: 3

DISCUSSION: Owens and Thordardson cautioned surgeons not to release the conjoined tendon
from the base of the proximal phalanx of the great toe because of an increased risk of iatrogenic
hallux varus. Release of the transverse and oblique heads of the adductor hallucis is largely
accomplished by releasing the soft tissue adjacent to the lateral sesamoid, without releasing tissue
from the base of the proximal phalanx. The conjoined tendon is made up of the flexor hallucis
brevis and the adductor hallucis.

REFERENCES: Owens S, Thordardson DB: The adductor hallucis revisited. Foot Ankle Int
2001;22:186-191.
Sarrafian SK: Anatomy of the Foot and Ankle. Philadelphia, PA, JB Lippincott, 1983, chapter 5.

64. Figures a and b show a clinical photograph and radiographs of a patient who sustained an
open calcaneus fracture in a motor vehicle accident. The patient received immediate IV
antibiotics and an emergent irrigation and debridement. The swelling has subsided by 3
weeks and the medial wound is clean. What do you tell the patient about the likelihood of
infection if a formal open reduction and internal fixation via a lateral approach is performed?

1- There is no significant difference between the infection rate for this fracture and a
similar closed fracture.
2- Due to the risk of infection, open reduction and internal fixation is not recommended
for this fracture.
3- The infection rate is three to five times more likely with this fracture.
4- Due to the risk of infection from a lateral approach, treatment is confined to limited
internal fixation or an external fixator.
5- The patient will need to undergo 3 weeks of IV antibiotics at home.

PREFERRED RESPONSE: 1

DISCUSSION: Multiple authors have shown similar infection rates for grade 1 and 2 open medial
fractures and closed fractures that have been treated with an extensile lateral approach and open
reduction and internal fixation. Patients only need IV antibiotics for 2 to 3 days after surgery.
Formal open reduction and internal fixation is not recommended for grade 3 medial wounds and
most lateral wounds.

REFERENCES: Heier KA, Infante AF, Walling AK, et al: Open fractures of the calcaneus: Soft-
tissue injury determines outcome. J Bone Joint Surg Am 2003;85:2276-2282.
Buckley RE, Tough S: Displaced intra-articular calcaneal fractures. J Am Acad Orthop Surg
2004;12:172-178.
65. When compared to traditional open repair through a posterior incision, percutaneous Achilles
tendon repair clearly results in a reduction of what complication?

1- Wound infection
2- Sural nerve injury
3- Achilles tendon re-rupture
4- Weakness
5- Deep venous thrombosis

PREFERRED RESPONSE: 1

DISCUSSION: Prospective studies, including randomized and randomized multicenter reports, have
shown that percutaneous or mini-open acute Achilles tendon repair has comparable functional results
when compared to traditional open techniques. Calder and Saxby reported one superficial infection
out of 46 patients with a mini-open repair; Assal and associates and Cretnik and associates had no
wound complications or infections. The other complications have not proved to be less likely with
the mini-open or percutaneous technique.

REFERENCES: Assal M, Jung M, Stern R, et al: Limited open repair of Achilles tendon ruptures:
A technique with a new instrument and findings of a prospective multicenter study. J Bone Joint
Surg Am 2002;84:161-170.
Calder JD, Saxby TS: Early, active rehabilitation following mini-open repair of Achilles tendon
rupture: A prospective study. Br J Sports Med 2005;39:857-859.
Cretnik A, Kosanovic M, Smrkolj V: Percutaneus versus open repair of the ruptured Achilles tendon:
A comparative study. Am J Sports Med 2005;33:1369-1379.

66. A 24-year-old woman was struck by a mini van in a parking lot and sustained a closed
segmental tibia fracture that was treated with an intramedullary nail the following morning.
Follow-up examinations reveal a slowly progressive clawing of all five toes, a progressive
equinocavovarus contracture, and the patient is unable to perform a single heel rise on the
affected limb. At 1 year after surgery, the patient now has a 10-degree equinus contracture
that is not relieved with knee flexion. Treatment should now consist of

1- physical therapy and bracing.


2- reassurance that these problems will resolve with time.
3- posterior capsule release, Achilles tendon lengthening, and excision of the scarred
muscle and tendon in the leg and foot.
4- Achilles tendon lengthening, and flexor digitorum longus and flexor hallucis longus
tenotomies at the individual digits with transfer of the posterior tibial tendon to the
dorsum of the foot.
5- flexor digitorum longus and flexor hallucis longus tenotomies at the individual digits
with midfoot capsular release and hallux interphalangeal fusion.
PREFERRED RESPONSE: 3

DISCUSSION: This is an example of a missed deep posterior compartment syndrome that typically
presents 6 months after the injury with progressive clawing due to necrosis, scarring, and contracture
of the posterior tibial tendon, flexor digitorum longus, and flexor hallucis longus. Treatment consists
of debridement of necrotic muscle and scar tissue with corresponding tendon excision. After
debridement and posterior capsule release, if the equinus is relieved with knee flexion, a
gastrocnemius slide may be performed. Otherwise, the lengthening should be at the level of the
Achilles tendon. Bracing will not address the claw toes.

REFERENCES: Hansen ST Jr: Functional Reconstruction of the Foot and Ankle. Philadelphia, PA,
Lippincott Williams & Wilkins, 2000, pp 212-213.
Manoli A II, Smith DG, Hansen ST Jr: Scarred muscle excision for the treatment of established
ischemic contracture of the lower extremity. Clin Orthop Relat Res 1993;292:309-314.
Early JS, Ricketts DS, Hansen ST: Treatment of compartmental liquefaction as a late sequelae of a
lower limb compartment syndrome. J Orthop Trauma 1994;8:445-448.

67. A 26-year-old man with chronic lateral ankle instability underwent a modified Brostrm
procedure 8 months ago. He reports persistent pain and swelling of the lateral ankle.
Examination reveals lateral ankle tenderness and swelling and a negative anterior drawer test.
Laboratory studies show a WBC count of 6,500/mm3 and an erythrocyte sedimentation rate
of 15 mm/h. Radiographs of the ankle are normal. What is the most likely cause of this
problem?

1- Deep infection
2- Failure of repair
3- Peroneus longus tear
4- Peroneus brevis tear
5- Tibiotalar arthritis

PREFERRED RESPONSE: 4

DISCUSSION: Chronic lateral instability is commonly associated with a longitudinal split tear of
the peroneus brevis tendon. The interrelationship of lateral ankle instability with superior retinacular
laxity and resultant peroneus brevis split can account for persistent lateral ankle pain in this patient.
Surgical treatment must identify and correct the underlying tendon pathology and should attempt to
repair or debride the peroneus brevis tendon, reconstruct the superior peroneal retinaculum, flatten
the posterior edge of the fibula by removing the sharp bony prominence, or deepening the fibular
groove, along with addressing lateral ankle ligamentous instability. The laboratory values are not
consistent with infection. A negative anterior drawer test confirms stability of the repair. Ankle
arthritis is not seen on radiographs and usually takes longer than 3 months to develop.
REFERENCES: Bonnin M, Tavernier T, Bouysset M: Split lesions of the peroneus brevis tendon in
chronic ankle laxity. Am J Sports Med 1997;25:699-703.
Sobel M, Geppert MJ, Warren RF: Chronic ankle instability as a cause of peroneal tendon injury.
Clin Orthop Relat Res 1993;296:187-191.
68. A 52-year-old woman with a 2-year history of a flexible (stage II) adult-acquired flatfoot
deformity has failed to respond to nonsurgical management consisting of immobilization,
custom orthotics, nonsteroidal anti-inflammatory drugs, and physical therapy. The patient is
unable to perform a single limb heel rise. Weight-bearing radiographs are shown in Figures a
through c. What is the most appropriate surgical correction?

1- Tendon transfer, lateral column lengthening, and heel cord lengthening


2- Triple arthrodesis and heel cord lengthening
3- Tendon transfer, lateral column lengthening, medial column arthrodesis, and heel cord
lengthening
4- Tendon transfer, spring ligament repair, and heel cord lengthening
5- Tendon repair, medial displacement calcaneal osteotomy, and heel cord lengthening

PREFERRED RESPONSE: 3

DISCUSSION: The patient has an atypical adult flatfoot deformity. The radiographs reveal forefoot
abduction, mild loss of calcaneal pitch, and marked plantar flexion sag through the
naviculocuneiform joint. The inability to perform a single limb heel rise indicates that the posterior
tibial tendon is nonfunctional; however, the deformity remains flexible. In this patient, surgical
treatment should include a tendon transfer, lateral column lengthening, medial column arthrodesis,
and heel cord lengthening. Because a substantial portion of the deformity stems from the
naviculocuneiform joint in this instance, tendon transfer and lateral column lengthening alone
provide insufficient deformity correction. Triple arthrodesis and heel cord lengthening is best
reserved for fixed flatfoot deformities. Soft-tissue procedures alone are associated with a high
failure rate, as are attempted tendon repairs.

REFERENCES: Greisberg J, Assal M, Hansen ST Jr, et al: Isolated medial column stabilization
improves alignment in adult-acquired flatfoot. Clin Orthop Relat Res 2005;435:197-202.
Greisberg J, Hansen ST Jr, Sangeorzan BJ: Deformity and degeneration in the hindfoot and midfoot
joints of the adult acquired flatfoot. Foot Ankle Int 2003;24:530-534.
69. Optimal management of the injury shown in Figure should include which of
the following?

1- Cast immobilization in equinus


2- Open reduction and internal fixation once the acute soft-tissue swelling has resolved
3- Urgent reduction and fixation
4- Arthroscopic-assisted percutaneous fixation
5- Open reduction and internal fixation with primary subtalar arthrodesis

PREFERRED RESPONSE: 3

DISCUSSION: The radiograph shows a displaced calcaneal beak fracture, a tongue-type fracture
variant. The fracture fragment typically includes the insertion point of the Achilles tendon, which
places marked tension on the thin overlying soft-tissue envelope and can lead to full-thickness
necrosis if not acutely addressed. Cast immobilization does not adequately address the increased
soft-tissue tension, as the fragment will be difficult to control. Arthroscopic-assisted techniques or
primary arthrodesis are not indicated because calcaneal beak fractures are typically extra-articular.

REFERENCES: Sanders RW, Clare MP: Fractures of the calcaneus, in Coughlin MJ, Mann RA,
Saltzman CL (eds): Surgery of the Foot and Ankle, ed 8. Philadelphia, PA, Mosby-Elsevier, 2007,
vol 2, pp 2017-2073.
Sanders RW, Clare MP: Fractures of the calcaneus, in Bucholz RW, Heckman JD, Court-Brown C
(eds): Rockwood and Greens Fractures in Adults, ed 6. Philadelphia, PA, Lippincott Williams &
Wilkins, 2006, vol 2, pp 2293-2336.
70. A 23-year-old man who was the restrained driver in a car involved in a high-speed motor
vehicle accident sustained the closed injury shown in Figures a through c. Which of the
following factors has the greatest impact on the risk of osteonecrosis?

1- Surgical stabilization within 6 to 8 hours of injury


2- Extent of initial fracture displacement
3- Nicotine use
4- Posterior-to-anterior screw fixation
5- Anatomic fracture reduction

PREFERRED RESPONSE: 2

DISCUSSION: The incidence of osteonecrosis following displaced talar neck fractures is most
related to the extent of initial fracture displacement. With increasing fracture displacement, the
tenuous vascular supply to the talar body is more at risk for damage, thereby increasing the risk of
osteonecrosis. Although displaced talar neck fractures have historically been considered a surgical
emergency, recent studies have shown that the timing of surgical intervention bears no impact on the
development of osteonecrosis. While nicotine use has an influence on fracture healing, it has never
been shown to be a factor in osteonecrosis, nor has posterior-to-anterior screw fixation or the quality
of fracture reduction.

REFERENCES: Lindvall E, Haidukewych G, Dipasquale T, et al: Open reduction and stable fixation
of isolated, displaced talar neck and body fractures. J Bone Joint Surg Am 2004;86:2229-2234.
Vallier HA, Nork SE, Barei DP, et al: Talar neck fractures: Results and outcomes. J Bone Joint Surg
Am 2004;86:1616-1624.

71. A 30-year-old woman injured her ankle playing soccer 3 months


ago. She now reports popping and pain over the lateral side of her
ankle. An MRI scan is shown in Figure. What structure needs to be
repaired to alleviate the popping?
1- Peroneal longus tendon
2- Peroneal brevis tendon
3- Superior peroneal retinaculum
4- Anterior talofibular ligament
5- Calcaneofibular ligament

PREFERRED RESPONSE: 3

DISCUSSION: The symptoms and MRI scan indicate dislocated peroneal tendons. In this patient,
the structure that needs to be repaired is the superior peroneal retinaculum. If the popping was
coming from a torn peroneal tendon, repair would involve the peroneal longus or brevis tendon, but
this is not shown in the MRI scan. The anterior talofibular ligament or the calcaneofibular ligament
would need to be repaired if the patient had ankle instability due to an ankle sprain.

REFERENCES: Jones DC: Tendon disorders of the foot and ankle. J Am Acad Orthop Surg
1993;1:87-94.
Timins ME: MR imaging of the foot and ankle. Foot Ankle Clin 2000;5:83-101.

72. A 35-year-old woman with type 1 diabetes mellitus has been treated for the past 2 years at a
wound care center for persistent bilateral fifth metatarsal head ulcers. Management has consisted of
shoe wear modifications, treatment with multiple enzymatic ointments, and a fifth metatarsal head
resection on the left side. Physical examination reveals intact pulses, minimal ankle dorsiflexion,
neutral hindfoot, and a persistent ulcer under the fifth metatarsal heads. What treatment will best
help heal the ulcers?

1- Plastizote orthotics with a metatarsal pad and a cutout under the fifth metatarsal head
2- Hyperbaric oxygen and prolonged non-weight-bearing
3- A healing shoe that completely alleviates any weight bearing on the forefoot
4- A gastrocnemius release and supportive wound care
5- A transmetatarsal amputation

PREFERRED RESPONSE: 4

DISCUSSION: The patient likely has a significant Achilles contracture that causes her to always
bear more weight on her forefoot. A gastrocnemius recession takes the ankle out of plantar flexion
and she will be able to return to a normal gait and reduce the pressures on her forefoot. A forefoot
amputation is a salvage option. The other choices are appropriate; however, the patient has had this
problem for 2 years and she has already had multiple attempts at shoe wear modification.
REFERENCES: Laughlin RT, Calhoun JH, Mader JT: The diabetic foot. J Am Acad Orthop Surg
1995;3:218-225.
Aronow MS, Diaz-Doran V, Sullivan RJ, et al: The effect of triceps surae contracture force on
plantar foot pressure distribution. Foot Ankle Int 2006;27:43-52.

73. The hallucal sesamoids are held together by which of the following structures?

1- Intersesamoid ligament
2- Intermetatarsal ligament
3- Spring ligament
4- Plantar fascia
5- Flexor hallucis longus tendon

PREFERRED RESPONSE: 1

DISCUSSION: The two sesamoids of the metatarsophalangeal joint are embedded in the tendons of
the short flexor of the great toe. They are held together by the intersesamoid ligament and the
plantar plate, which inserts on the base of the proximal phalanx of the hallux. The flexor hallucis
longus tendon inserts onto the distal phalanx of the great toe. The plantar calcaneonavicular (spring)
ligament, by supporting the head of the talus, principally maintains the arch of the foot. The plantar
fascia inserts distally onto the skin and to the flexor tendons and transverse metatarsal ligaments at
each metatarsophalangeal joint. The intermetatarsal ligament attaches to the base of the second
through fifth metatarsals.

REFERENCES: Lewis WH (ed): Grays Anatomy of the Human Body, ed 20. Philadelphia, PA, Lea
& Febiger, 2000.
Richardson EG: Hallucal sesamoid pain: Causes and surgical treatment. J Am Acad Orthop Surg
1999;7:270-278.

74. Figures a and b show the clinical photograph and a weight-bearing radiograph of a patient
with diabetes mellitus who has had recurrent ulcers under the head of the talus that have
previously resolved with a series of non-weight-bearing total contact casts. The deformity
does not correct passively. Dorsalis pedis and posterior tibial pulses are palpable. The
patient is insensate to the Semmes-Weinstein 5.07 (10 gm) monofilament. The ulcer is
currently healed. What is the best option to prevent recurrent ulceration and infection?
1- Therapeutic footwear/depth-inlay shoes and a custom accommodative foot orthosis
2- Charcot restraint orthotic walker (CROW)
3- Percutaneous Achilles tendon lengthening, a total contact cast, and a CROW walker
4- Surgical correction of the deformity, Achilles tendon lengthening, and therapeutic
footwear
5- Ankle disarticulation (Syme) amputation

PREFERRED RESPONSE: 4

DISCUSSION: This is a nonplantigrade deformity in a patient with a Charcot foot deformity.


Longitudinal studies have shown that recurrent ulceration/infection is likely unless the deformity is
corrected. Achilles tendon lengthening is advised for simple forefoot ulcers. The current approach
to this problem is best managed with surgical correction of the deformity, Achilles tendon
lengthening, and therapeutic footwear.

REFERENCES: Bevan WP, Tomlinson MP: Radiographic measures as a predictor of ulcer formation
in diabetic charcot midfoot. Foot Ank Int 2008;29:568-573.
Simon SR, Tejwani SG, Wilson DL, et al: Arthrodesis as an early alternative to nonoperative
management of Charcot arthropathy of the diabetic foot. J Bone Joint Surg Am 2000;82:939-950.
Pinzur M: Surgical versus accommodative treatment for Charcot arthropathy of the midfoot.
Foot Ankle Int 2004;25:545-549.

75. Which of the following conditions precludes performing a tendon transfer?

1- The target joint has a full passive range of motion.


2- The range of motion of the target joint only occurs in the direction of correction.
3- The target joint cannot be passively corrected to its neutral position.
4- The muscle to be transferred is out-of-phase.
5- There is no pulley to assist the transferred muscles fulcrum.
PREFERRED RESPONSE: 3

DISCUSSION: Several conditions must be met before a tendon transfer has the potential to correct a
dynamic deformity. If the target joint cannot be passively corrected to neutral, it indicates that a
static joint contracture or bony deformity exists that cannot be corrected with a dynamic tendon
transfer. While in-phase muscles are best, out-of-phase muscles are often the only muscles available
for transfer. Tendon transfer should pull in a straight line to avoid tethering and late failure.

REFERENCES: Canale ST (ed): Campbells Operative Orthopaedics, ed 10. St Louis, MO, Mosby,
2003, pp 1283-1287.
Coughlin MJ, Mann RA: Disorders of tendons, in Coughlin MJ, Mann RA (eds): Surgery of the Foot
and Ankle, ed 7. St Louis, MO, Mosby, 1999, pp 786-861.

76. A 26-year-old rugby player injured his foot when tackled from behind. Radiographs are seen
in Figures a through c. What is the most appropriate treatment?

1- Closed reduction and percutaneous pin fixation


2- Application of a short leg non-weight-bearing cast
3- Application of a walking boot with weight bearing as tolerated
4- Open reduction and internal fixation
5- Elastic bandage wrap and activity as tolerated

PREFERRED RESPONSE: 4

DISCUSSION: The patient has a ligamentous Lisfranc injury. Diastasis seen between the bases of
the second metatarsal and medial cuneiform is pathognomonic for a rupture of the Lisfrancs
ligament. This injury is best treated surgically with either open reduction and internal fixation or
possibly closed manipulation and percutaneous screw fixation if anatomic alignment can be achieved
closed. Pin fixation has been shown to be inferior to screw fixation due to the length of time that
fixation is required for adequate ligament healing.
REFERENCES: Chiodo CP, Myerson MS: Developments and advances in the diagnosis and
treatment of injuries of the tarsometatarsal joint. Orthop Clin North Am 2001;32:11-20.
Hunt SA, Ropiak C, Tejwani NC: Lisfranc joint injuries: Diagnosis and treatment. Am J Orthop
2006;35:376-385.

77. A 32-year-old woman sustained a closed calcaneus


fracture 2 years ago and was treated nonsurgically. She
now reports a 6-month history of progressively worsening
pain over the anterior ankle and lateral hindfoot.
Climbing stairs and ascending slopes is particularly
difficult for her. Bracing and intra-articular corticosteroid
injections have not provided sufficient relief. Figure
shows a weight-bearing lateral radiograph. What is the
most appropriate surgical option?

1- Subtalar arthrodesis in situ with plantar flexion osteotomy of the talar neck
2- Distraction subtalar arthrodesis with a corticocancellous bone block autograft
3- Subtalar arthrodesis in situ
4- Triple arthrodesis
5- Subtalar arthrodesis in situ with anterior ankle exostectomy

PREFERRED RESPONSE: 2

DISCUSSION: Intra-articular fractures of the calcaneus often include depression of the posterior
facet of the subtalar joint. This can lead to dorsiflexion of the talus because of diminished height
posteriorly. In a weight-bearing position, the dorsal surface of the talar neck can impinge against the
distal tibia, causing anterior ankle pain. In addition, posttraumatic arthritis of the subtalar joint
typically occurs after a calcaneus fracture. The patients symptoms are consistent with both anterior
ankle impingement and subtalar degenerative arthritis. The Bohler angle, approximately 15 degrees,
confirms depression of the posterior facet. Distraction subtalar arthrodesis with a corticocancellous
bone block autograft will improve talar declination, decrease anterior impingement, and address the
subtalar degenerative arthritis simultaneously.

REFERENCES: Rammelt S, Grass R, Zawadski T, et al: Foot function after subtalar distraction
bone-block arthrodesis: A prospective study. J Bone Joint Surg Br 2004;86:659-668.
Trnka HJ, Easley ME, Lam PW, et al: Subtalar distraction bone block arthrodesis. J Bone Joint Surg
Br 2001;83:849-854.
78. A 42-year-old woman who observes traditional Muslim practices is seen in your office
accompanied by her physician husband to discuss possible elective bunion correction. In
considering the treatment of this patient, what is one of the most important considerations?

1- The role her husband will play in the decision to proceed with surgery
2- Her role as primary caregiver in the household
3- Dietary concerns during her hospitalization
4- Daily cleansing rituals that may affect wound care
5- The importance of maintaining modesty precautions during examination, surgery, and
postoperative appointments

PREFERRED RESPONSE: 5

DISCUSSION: In considering faith-based issues regarding treatment of this patient, the presence of
her husband for the office visit would imply an agreement with her decision to have surgery. It also
may facilitate her examination. Her role as caregiver, dietary concerns, and cleansing rituals are less
important considerations with an outpatient-based procedure. Privacy concerns remain paramount to
Muslim women, which include limited exposure during examination, during surgery, and in
subsequent follow-up visits.

REFERENCE: Jimenez R, Lewis VO (eds): Culturally Competent Care Guidebook. Rosemont, IL,
American Academy of Orthopaedic Surgeons, 2007.

79. A 35-year-old female runner reports progressive vague aching pain involving her midfoot.
Her pain is most notable when running. She denies specific injury. Examination reveals
minimal swelling and localized tenderness over the dorsal medial midfoot and navicular.
Radiographs and an MRI scan are shown in Figures a through c. What is the most
appropriate management?

1- Non-weight-bearing immobilization and CT


2- Walking boot and weight bearing as tolerated
3- Activity restrictions (avoidance of running) and repeat radiographs in 2 to 4 weeks
4- Activity restrictions and a bone scan
5- Surgical fixation
PREFERRED RESPONSE: 1

DISCUSSION: A high index of suspicion is required to identify a possible navicular stress fracture,
especially in runners. High pain tolerance in the competitive athlete and often minimal swelling
contribute to frequent delays in diagnosis. Localized tenderness over the dorsal navicular (so-called
N spot) in a running athlete should alert the treating physician. In this patient, the radiographs are
negative and the MRI scan shows marrow edema within the navicular. This could represent a stress
reaction, stress fracture, or osteonecrosis. Appropriate management should include non-weight-
bearing immobilization and obtaining a CT scan to determine if a fracture is present. Early surgical
treatment may be considered but only if a fracture is identified.

REFERENCES: Lee A, Anderson R: Stress fractures of the tarsal navicular. Foot Ankle Clin
2004;9:85-104.
Coughlin M: Tarsal navicular stress fractures. Tech Foot Ankle Surg 2002;1:112-122.

80. A 47-year-old woman underwent a bunionectomy and hallux valgus correction a few years
ago. She now has the complication shown in Figures a and b. She has no pain with motion
of the metatarsophalangeal or interphalangeal joints. What is the best reconstructive option
in this setting?

1- Metatarsophalangeal joint arthrodesis


2- Medial capsular release with lengthening of the abductor hallucis
3- Medial capsular release with lateral sesamoid excision
4- Proximal phalangeal lateral closing wedge osteotomy
5- Medial capsular lengthening and split extensor hallucis longus tendon transfer

PREFERRED RESPONSE: 5
DISCUSSION: The patient has a flexible hallux varus that is a complication of the bunion surgery.
With joints that are not arthritic and still flexible, a medial release is necessary to realign the joint.
The extensor hallucis longus split transfer helps maintain position and still preserve motion at the
interphalangeal joint level. Arthrodesis is a salvage procedure. Soft-tissue releases alone are most
likely inadequate. Excision of the lateral sesamoid is contraindicated because that further
compromises the forces resisting hallux varus. Phalangeal osteotomy would not address the medial
subluxation at the metatarsophalangeal joint.

REFERENCES: Richardson EG (ed): Orthopaedic Knowledge Update: Foot and Ankle 3.


Rosemont, IL, American Academy of Orthopaedic Surgeons, 2004, pp 27-32.
Coughlin MJ, Mann RA, Saltzman CL (eds): Surgery of the Foot and Ankle, ed 8. Philadelphia, PA,
Mosby, 2007, pp 345-351.
81. A 43-year-old man reports a 3-year history of progressively worsening
pain in the first metatarsophalangeal joint that is aggravated by activity.
Larger shoes, intra-articular corticosteroid injections, and a Mortons
extension pedorthic have failed to provide relief. Motion is limited to 10
degrees of dorsiflexion, and the grind test is positive. An AP
radiograph is shown in Figure. What is the most appropriate surgical
treatment?

1- Cheilectomy
2- Moberg osteotomy
3- Keller resection arthroplasty
4- Resurfacing implant hemiarthroplasty
5- First metatarsophalangeal arthrodesis

PREFERRED RESPONSE: 5

DISCUSSION: Stage III hallux rigidus comprises end-stage degenerative arthritis with loss of
cartilage from the phalanx and metatarsal. Therefore, cheilectomy, osteotomy, and resection
arthroplasty are inadequate. Resection arthroplasty results in diminished propulsion and transfer
metatarsalgia. Resurfacing implant hemiarthroplasty remains unproven for earlier stages of hallux
rigidus, but is not appropriate when there is cartilage loss from the base of the proximal phalanx.
First metatarsophalangeal arthrodesis has proven to be a very reliable and functional treatment of
end-stage hallux rigidus.

REFERENCES: Gibson JN, Thomson CE: Arthrodesis or total replacement arthroplasty for hallux
rigidus: A randomized controlled trial. Foot Ankle Int 2005;26:680-690.
Brage ME, Ball ST: Surgical options for salvage of end-stage hallux rigidus. Foot Ankle Clin
2002;7:49-73.

82. A 12-year-old girl who plays softball has chronic lateral hindfoot aching pain that is
aggravated by weight-bearing activity. She reports that the pain has recurred after initial
improvement with cast immobilization, and it continues to limit her overall level of activity.
Radiographs are seen in Figures a through c. What is the most appropriate surgical
treatment?
1- Correction of the flatfoot deformity
2- Achilles tendon lengthening followed by orthotic support
3- Excision of the tarsal coalition
4- Sinus tarsi debridement
5- Triple arthrodesis

PREFERRED RESPONSE: 3

DISCUSSION: The patient has a calcaneonavicular tarsal coalition. Symptoms of calcaneonavicular


coalitions typically are seen between the ages of 10 and 14 years. The cause of pain has not been
clearly established. It has been postulated that the coalition stiffens with maturity and microfractures
can result, producing pain. Resection of a calcaneonavicular coalition generally has been associated
with a satisfactory result. Soft-tissue interposition, most commonly using the extensor digitorum
brevis muscle, appears to be helpful. A hindfoot arthrodesis (usually triple) would be reserved if
coalition resection proves to be unsuccessful. Achilles tendon lengthening and orthotic support, as
well as debridement of the sinus tarsi, are not expected to result in a satisfactory outcome. The
patient does not have a flatfoot deformity.

REFERENCES: Vaccaro AR (ed): Orthopaedic Knowledge Update 8. Rosemont, IL, American


Academy of Orthopaedic Surgeons, 2005, pp 757-765.
Lemley F, Berlet G, Hill K, et al: Current concepts review: Tarsal coalition. Foot Ankle Int
2006;27:1163-1169.
83. A 38-year-old man underwent a transtibial amputation for chronic posttraumatic foot and
ankle pain and chronic calcaneal osteomyelitis. Postoperative radiographs are seen in
Figures a and b. What is the proposed purpose of the surgical modification seen in the
radiographs?

1- Reduces shrinkage of the residual limb


2- Creates a more stable platform for load transfer
3- Reduces wound healing complications by avoiding the soft-tissue dissection
necessary to transect the fibula at a level proximal to the tibia
4- Connecting bone strut provides an attachment point for more effective myodesis
5- Allows a more proximal resection level to decrease tension on the wound

PREFERRED RESPONSE: 2

DISCUSSION: The Ertl modification of a below-knee amputation has been proposed to create a
more stable platform to aid in transferring the load of weight bearing between the residual limb
and the prosthetic socket. It is felt that a stable platform allows total contact loading over an
enlarged stable surface area. Early studies have suggested that this modification may enhance the
patients perceived functional outcome.

REFERENCES: Pinzur MS, Pinto MA, Saltzman M, et al: Health-related quality of life in patients
with transtibial amputation and reconstruction with bone bridging of the distal tibia and fibula.
Foot Ankle Int 2006;27:907-912.
Pinzur MS, Pinto MA, Schon LC, et al: Controversies in amputation surgery. Instr Course Lect
2003;52:445-451.

84. Figures a through c show the clinical photographs and radiograph of a patient with diabetes
mellitus who lives independently. The patient was admitted to the hospital late yesterday
afternoon with clinical signs of sepsis. Parenteral antibiotic therapy resolved the sepsis, and
blood glucose levels are now well controlled. The patient has no palpable pulses. The ankle-
brachial index is 0.70. Laboratory studies show a WBC count of 8,500/mm3, a serum
albumin of 1.9 g/dL, and a total lymphocyte count of 1,500/mm3. What treatment has the
best potential to optimize his survival and independence?

1- Local wound care, parenteral antibiotic therapy, metabolic support, and reevaluation
in 1 week
2- Vascular consultation for a bypass operation
3- Syme ankle disarticulation
4- Guillotine transtibial amputation
5- Closed transtibial amputation

PREFERRED RESPONSE: 1

DISCUSSION: The patient was admitted to the hospital with sepsis. The sepsis has resolved,
leaving the patient with a negative nitrogen balance. Now that the patient is stable, metabolic
support should be used to optimize his nutrition. If the serum albumin can be increased to 2.5 g/dL,
he has an excellent potential to heal an amputation at the Syme ankle disarticulation level; a level
that will optimize his functional independence.

REFERENCES: Pinzur MS, Stuck RR, Sage R, et al: Syme ankle disarticulation in patients with
diabetes. J Bone Joint Surg Am 2003;85:1667-1672.
Pinzur MS, Smith D, Osterman H: Syme ankle disarticulation in peripheral vascular disease and
diabetic foot infection: The one-stage versus two-stage procedure. Foot Ankle Int 1995;16:124-127.

85. A toddler is brought in by his parents for evaluation of gait problems. Birth history and
neurologic examination are unremarkable. After evaluating femoral torsion, tibial torsion,
and foot contour, the diagnosis is excessive internal tibial torsion. The parents should be
advised to expect which of the following outcomes?

1- Resolution by age 3 or 4 years without active treatment in most patients


2- Resolution by age 8 or 9 years without active treatment in most patients
3- Resolution with casting as the most effective treatment
4- Resolution with bracing and shoe modification as the most effective treatment
5- Resolution with surgery as the most effective treatment

PREFERRED RESPONSE: 1

DISCUSSION: Excessive internal tibial torsion is a common cause of intoeing in toddlers. In most
children, this resolves spontaneously by 3 to 4 years of age. Intoeing in elementary age children is
usually the result of excessive femoral anteversion. Studies have shown that active intervention
(casting, splinting, and shoe modifications) has no demonstrable effect on the natural history or
resolution of tibial torsion. Surgery is rarely indicated in adolescents with severe internal tibial
torsion that has not resolved and is resulting in cosmetic and functional problems.

REFERENCES: Canale ST, Beaty JH: Operative Pediatric Orthopaedics. St Louis, MO, Mosby
Year Book, 1991, pp 357-385.
Lincoln TL, Suen PW: Common rotational variations in children. J Am Acad Orthop Surg
2003;11:312-320.

86. Arthrodesis of which of the following joints has the greatest cumulative effect on
midfoot/hindfoot motion?

1- Talonavicular
2- Naviculocuneiform
3- Subtalar
4- Cuboid-fifth metatarsal
5- Calcaneocuboid

PREFERRED RESPONSE: 1

DISCUSSION: Arthrodesis of the talonavicular joint eliminates almost all hindfoot motion.
Arthrodesis of the subtalar joint eliminates 74% of talonavicular motion and 44% of calcaneocuboid
motion. Arthrodesis of the calcaneocuboid joint eliminates 33% of talonavicular motion and 8% of
subtalar motion. Arthrodesis of the naviculocuneiform or cuboid-fifth metatarsal joint has limited
effect on hindfoot motion.

REFERENCES: Astion DJ, Deland JT, Otis JC, et al: Motion of the hindfoot after simulated
arthrodesis. J Bone Joint Surg Am 1997;79:241-246.
Savory KM, Wlker N, Stukenborg C, et al: Biomechanics of the hindfoot joints in response to
degenerative hindfoot arthrodeses. Clin Biomech 1998;13:62-70.

87. A 51-year-old man sustained an open fracture of his tibia in Korea 42 years ago. An
infection developed and it was resolved with surgical treatment. For the past 6 months, an
ulcer with mild drainage has developed over the medial tibia. The ulcer is small and there is
minimal erythema at the ulcer site. A radiograph and MRI scan are shown in Figures a and
Figure b. Initial cultures show Staphylococcus aureus susceptible to the most appropriate
antibiotics. Laboratory studies show an erythrocyte sedimentation rate of 70 mm/h. What is
the most appropriate surgical treatment at this time?

1- Irrigation and debridement of the cystic lesion and 6 weeks of IV antibiotics


2- Curettage, debridement of nonviable bone, and placement of absorbable antibiotic
beads, followed by a course of IV antibiotics from 1 to 4 weeks and a 6-week course
of oral antibiotics
3- Complete resection of the infected portion of bone, placement of an external fixator to
stabilize the tibia, and 6 weeks of IV antibiotics
4- Amputation
5- Local debridement of bone and the overlying skin and soft tissues, 6 weeks of IV
antibiotics, and free-flap wound coverage

PREFERRED RESPONSE: 2

DISCUSSION: The patient has chronic tibial osteomyelitis that is due to low virulent bacteria. The
history and studies do not suggest the need for an amputation or a free-flap procedure. This is a
localized tibial infection that is in a healed bone; there is no need to resect the entire area of the tibia
bone around the infection. The most appropriate treatment is curettage, debridement of nonviable
bone, and placement of absorbable antibiotic beads, followed by a course of IV antibiotics from 1 to
4 weeks and a 6-week course of oral antibiotics. Studies have shown that in cases of localized
osteomyelitis that are of low virulence, as little as 1 week of IV antibiotics followed by 6 weeks of
oral antibiotics is successful.

REFERENCES: Patzakis MJ, Zalavras CG: Chronic posttraumatic osteomyelitis and infected
nonunion of the tibia: Current management concepts. J Am Acad Orthop Surg 2005;13:417-427.
Beals RK, Bryant RE: The treatment of chronic open osteomyelitis of the tibia in adults.
Clin Orthop Relat Res 2005;433:212-217.

88. Which of the following best describes the relationship of the anterior tibial artery and dorsalis
pedis artery to the extensor hallucis longus (EHL) tendon as they progress from the level of
the ankle to the dorsum of the foot?
1- Artery medial, then lateral
2- Artery lateral, then medial
3- Artery always medial
4- Artery always lateral
5- Artery always deep

PREFERRED RESPONSE: 1

DISCUSSION: At the ankle level, the anterior tibial artery lies medial to the EHL tendon. The
artery becomes the dorsalis pedis after crossing onto the dorsum of the foot. At this point, the artery
lies lateral to the tendon.

REFERENCES: Resch S: Functional anatomy and topography of the foot and ankle, in Myerson M
(ed): Foot and Ankle Disorders. Philadelphia, PA, WB Saunders, 2000, vol 1, pp 25-49.
Sarrafian SK: Anatomy of the Foot and Ankle: Descriptive, Topographic, Functional, ed 2.
Philadelphia, PA, JB Lippincott, 1993.

89. A 42-year-old man reports a 12-month history of a painful fusiform swelling of the Achilles
tendon. Physical therapy, heel lifts, and anti-inflammatory drugs have failed to provide relief. MRI
scans are shown in Figures a and b. What is the treatment of choice?

1- Steroid injection
2- Debridement and side-to-side repair
3- Debridement and flexor hallucis longus tendon transfer
4- Brisement
5- Continued nonsurgical management with use of a short leg walking cast

PREFERRED RESPONSE: 3

DISCUSSION: The area of the tendon degeneration is greater than 50% of the width so a
supplemental tendon transfer is needed. Debridement and repair alone do not provide adequate
strength. Injection risks tendon rupture. Brisement is indicated for peritendinitis, not tendinosis.
Nonsurgical management is unlikely to be of benefit after 12 months.

REFERENCES: Richardson EG (ed): Orthopaedic Knowledge Update: Foot and Ankle 3.


Rosemont, IL, American Academy of Orthopaedic Surgeons, 2004, pp 94-95.
Den Hartog BD: Flexor hallucis longus transfer for chronic Achilles tendinosis. Foot Ankle Int
2003;24:233-237.

90. A 35-year-old man is seen for evaluation of his left


ankle following multiple previous ankle sprains
and frequent episodes of the ankle giving way.
Examination reveals marked laxity about the
lateral ankle with associated tenderness along the
peroneal tendons. Physical therapy, anti-
inflammatory drugs, and supportive bracing have
failed to provide relief. An MRI scan shows
peroneal tenosynovitis and a possible tear. He
elects to undergo a peroneal tendon repair and
lateral ligament reconstruction. Which of the
following best describes the structure labeled A
in Figure?
1- Longitudinal split tear in the peroneus longus
2- Longitudinal split tear in the peroneus brevis
3- Plantaris
4- Peroneus accessorius
5- Peroneus quartus

PREFERRED RESPONSE: 5
DISCUSSION: The structure labeled A is a peroneus quartus, a supernumary muscle arising most
commonly from the peroneus brevis. The presence of peroneus quartus is not uncommon, with an
incidence of up to 21%, and is associated with lateral ankle pain and peroneal tendon symptoms,
theoretically as a result of mass effect within the peroneal tendon sheath.

REFERENCES: Zammit J, Singh D: The peroneus quartus muscle: Anatomy and clinical relevance.
J Bone Joint Surg Br 2003;85:1134-1137.
Sobel M, Levy ME, Bohne WH: Congenital variations of the peroneus quartus muscle: An anatomic
study. Foot Ankle 1990;11:81-89.

91. You are asked to evaluate the patient whose current clinical photographs are shown in
Figures a and b following aortic valve replacement 9 days ago. He is currently taking
anticoagulation medication. He has no systemic signs of sepsis. What is the best
management?

1- Warming in a water or saline bath at 104 degrees F (40 degrees C) with no dressings
2- Observation with possible late debridement
3- Urgent arteriogram
4- Reversal of anticoagulation, parenteral antibiotics, and debridement
5- Reversal of anticoagulation, parenteral antibiotics, and open forefoot amputation

PREFERRED RESPONSE: 2

DISCUSSION: These lesions are emboli related to the cardiac surgery, and the patient is already on
anticoagulation medication. The foot reveals no signs consistent with gangrene or infection. Unless
the patient shows local or systemic signs of sepsis, the best management is observation. It is
unlikely that formal debridement will be necessary.

REFERENCES: Bowker JH, Pfeiffer MA (eds): The Diabetic Foot. St Louis, MO, Mosby, 2001,
pp 219-260.
Coughlin MJ, Mann RA: Soft tissue disorders of the foot, in Coughlin MJ, Mann RA (eds): Surgery
of the Foot and Ankle, ed 7. St Louis, MO, Mosby, 1999, pp 1373-1397.
92. A 48-year-old woman with a history of a spinal cord injury as a teenager, has unilateral
weakness in the left lower extremity. She has used an ankle-foot orthosis for many years
without difficulty but recently has had a recurrent painful callus beneath the great toe that has
been recalcitrant to nonsurgical management. Examination reveals intact sensation with an
intractable plantar keratosis (IPK) beneath the first metatarsal head. Motor examination
reveals no active ankle or great toe dorsiflexion, and 4/5 plantar flexion strength at the ankle
and great toe. Passive ankle dorsiflexion is 10 degrees, whereas passive plantar flexion is
40 degrees. Passive great toe dorsiflexion is 30 degrees and plantar flexion is 10 degrees.
Foot alignment on standing is normal. Radiographs are shown in Figures a and b with a
marker beneath the IPK. Based on her request for surgical treatment, what is the most
appropriate procedure?

1- Gastrocnemius recession
2- Keller bunionectomy
3- Flexor hallucis longus tendon transfer
4- Planing/excising the superficial half of the medial sesamoid
5- Dorsiflexion osteotomy of the first metatarsal

PREFERRED RESPONSE: 4

DISCUSSION: Passive dorsiflexion is adequate to accommodate standing erect without excessive


pressure, and a gastrocnemius recession may lead to more instability. Complete excision of the
medial sesamoid could lead to an iatrogenic hallux valgus deformity. She does not have a cock-up
toe deformity; therefore, a flexor hallucis longus tendon transfer is not warranted. There is no
significant foot deformity; therefore, a dorsiflexion osteotomy is not warranted. The appropriate
procedure is planing of the plantar half of the medial sesamoid, thereby preserving its function while
diminishing the excessive pressure.

REFERENCES: Grace DL: Sesamoid problems. Foot Ankle Clin 2000;5:609-627.


Mizel MS, Miller RA, Scioli MW (ed): Orthopaedic Knowledge Update: Foot and Ankle 2.
Rosemont, IL, American Academy of Orthopaedic Surgeons, 1998, pp 135-150.
Richardson EG: Hallucal sesamoid pain: Causes and surgical treatment. J Am Acad Orthop Surg
1999;7:270-278.
93. The cavovarus deformity associated with Charcot-Marie-Tooth (CMT) disease is caused by
which of the following?

1- Streptococcal disease during infancy


2- Viral infection of the motor nerves
3- Sex-linked selective motor imbalance
4- Autosomal-dominant myelin sheath disease
5- Germ cell defect leading to asymmetrical growth disturbance

PREFERRED RESPONSE: 4

DISCUSSION: The most common inherited neuromuscular disease seen by orthopaedic surgeons is
CMT, which is an inherited autosomal-dominant disease. It is more commonly seen in men due to
the nature of the inheritance. Identification of cavus deformity in the foot of a child should arouse
suspicion.

REFERENCES: Richardson EG (ed): Orthopaedic Knowledge Update: Foot and Ankle 3.


Rosemont, IL, American Academy of Orthopaedic Surgeons, 2004, pp 135-143.
Charcot-Marie-Tooth Disease (CMT) Penn State Hershey Medical Center.
www.hmc.psu.edu/healthinfo/c/cmt.htm

94. When performing a gastrocnemius recession, what structure should be protected?

1- Tibial nerve
2- Sural artery
3- Plantaris
4- Posterior tibial artery
5- Sural nerve

PREFERRED RESPONSE: 5

DISCUSSION: When performing a gastrocnemius slide at the tendinous portion of the


gastrocnemius insertion, the sural nerve and saphenous vein, which tend to run midline posterior at
this level, must be protected and retracted laterally. An anatomic study of the sural nerve at this level
localized the nerve superficial to the deep fascia overlying the gastrocnemius in 42.5% of the cases;
deep to the superficial fascia in 57.5% of the cases, and directly applied to the gastrocnemius tendon
in 12.5% of cases.

REFERENCES: Pinney SJ, Sangeorzan BJ, Hanen ST Jr: Surgical anatomy of the gastrocnemius
resection (Strayer procedure). Foot Ankle Int 2004;25:247-250.
Hansen ST Jr: Functional Reconstruction of the Foot and Ankle. Philadelphia, PA, Lippincott
Williams & Wilkins, 2000, pp 415-417.

95. A 59-year-old woman underwent open reduction and internal fixation


(ORIF) of her ankle
6 months ago, with subsequent hardware removal 3 months later. She
now reports persistent, diffuse ankle pain, swelling, and limited range
of motion. Figure shows an oblique radiograph of the ankle. What is
the next most appropriate step in management?

1- Physical therapy
2- Hardware removal
3- Repeat placement of a syndesmotic screw
4- Deltoid ligament reconstruction
5- Revision ORIF with exploration of the syndesmosis and medial ankle

PREFERRED RESPONSE: 5

DISCUSSION: The radiographs demonstrate persistent widening of the medial clear space with an
ossicle. This represents soft-tissue interposition-scar tissue, the deltoid ligament, or the posterior
tibialis tendon. Physical therapy will not improve the symptomatic malalignment. Hardware
removal would be indicated for pain localized to the lateral fibula. Repeat syndesmotic screw
fixation alone will not reduce the malalignment. Deltoid ligament repair may be necessary but will
need to be combined with debridement of the medial ankle and syndesmosis, as well as repeat
placement of one or more syndesmotic screws to maintain the reduction.

REFERENCES: Weening B, Bhandari M: Predictors of functional outcome following


transsyndesmotic screw fixation of ankle fractures. J Orthop Trauma 2005;19:102-108.
Harper MC: Delayed reduction and stabilization of the tibiofibular syndesmosis. Foot Ankle Int
2001;22:15-18.
96. A farmer is seen in the emergency department after falling out of a hay loft onto the barn
floor below. He is unable to bear weight. Exploration of a 0.5 cm laceration over the
anterior tibia reveals bone. Radiographs reveal oblique displaced midshaft tibial and fibular
fractures. Based on these findings, what is the most appropriate antibiotic prophylaxis?

1- Cephalosporin
2- Cephalosporin and aminoglycoside
3- Cephalosporin and penicillin
4- Cephalosporin and vancomyacin
5- Cephalosporin, aminoglycoside, and penicillin

PREFERRED RESPONSE: 5

DISCUSSION: A farm injury is automatically considered a grade III (Gustillo classification) injury
regardless of size, energy, or additional soft-tissue injury due to the likelihood of substantial
contamination. Antibiotic recommendations for grade III injuries include a first- or second-
generation cephalosporin with an aminoglycoside or fluoroquinolone within 3 hours of injury, with
penicillin added for farm injuries.

REFERENCES: Okike K, Bhattacharyya T: Trends in the management of open fractures: A critical


analysis. J Bone Joint Surg Am 2006;88:2739-2748.
Holtom PD: Antibiotic prophylaxis: Current recommendations. J Am Acad Orthop Surg
2006:14:S98-S100.

97. A 66-year-old patient with type 1 diabetes mellitus has a deep, nonhealing ulcer under the
first metatarsal head and a necrotic tip of the great toe. He has been under the direction of a
wound care clinic for 4 months, and has had orthotics and shoe wear changes. What
objective findings are indicative of the patients ability to heal the wound postoperatively?

1- Absolute toe pressures of 55 mm Hg


2- Transcutaneous oxygen level of 20 mm Hg
3- Arterial brachial indices (ABI) of 1.2 at the level of surgery
4- ABI 0.3 at the level of surgery
5- Albumin level of 2.5

PREFERRED RESPONSE: 1
DISCUSSION: Absolute toe pressures greater than 40 to 50 mm Hg are a good sign of healing
potential. An ABI of greater than 0.45 favors healing, but indices greater than 1 are falsely positive
due to calcifications in the vessels. Normal albumin is an overall indication of nutritional status.
A transcutaneous oxygen level should be greater than 40 mm Hg for healing.

REFERENCES: Mizel MS, Miller RA, Scioli MW (eds): Orthopaedic Knowledge Update: Foot and
Ankle 2. Rosemont, IL, American Academy of Orthopaedic Surgeons, 1998, pp 113-122.
Pinzur MS, Stuck R, Sage R: Benchmark analysis on diabetics at high risk for lower extremity
amputation. Foot Ankle Int 1996;17:695-700.

98. Which of the following have been found to affect the rate of perioperative infections or
wound complication rates in foot and ankle surgery?

1- Methotrexate
2- Gold
3- Hydroxychloroquine
4- TNF-a inhibitors
5- Smoking

PREFERRED RESPONSE: 5

DISCUSSION: Clinical studies have shown that smoking cessation for 4 weeks reduces the risk of
infection to the level of nonsmokers. Adverse effects on wound healing caused by chemotherapy
used to treat rheumatoid arthritis has not been borne out in the literature.

REFERENCES: Bibbo C, Anderson RB, Davis WH, et al: The influence of rheumatoid
chemotherapy, age, and presence of rheumatoid nodules on postoperative complications in
rheumatoid foot and ankle surgery: Analysis of 725 procedures in 104 patients. Foot Ankle Int
2003;24:40-44.
Bibbo C, Goldberg JW: Infections and healing complications after elective orthopaedic foot and
ankle surgery during tumor necrosis factor-alpha inhibition therapy. Foot Ankle Int
2004;25:331-335.
Sorensen LT, Karlsmark T, Gottrup F: Abstinence from smoking reduces incisional wound infection:
A randomized controlled trial. Ann Surg 2003;238:1-5.
99. Intrinsic muscles of the foot act on the toes by

1- abducting the metatarsophalangeal joints and flexing the interphalangeal joints.


2- extending the metatarsophalangeal and interphalangeal joints.
3- extending the metatarsophalangeal joints and flexing the interphalangeal joints.
4- flexing the metatarsophalangeal and interphalangeal joints.
5- flexing the metatarsophalangeal joints and extending the interphalangeal joints.

PREFERRED RESPONSE: 5

DISCUSSION: Intrinsic muscles of the foot function to flex the metatarsophalangeal joints and
extend the interphalangeal joints.

REFERENCES: Myerson MS, Shereff MJ: The pathologic anatomy of claw and hammertoes.
J Bone Joint Surg Am 1989;71:45-49.
Richardson EG (ed): Orthopaedic Knowledge Update: Foot and Ankle 3. Rosemont, IL, American
Academy of Orthopaedic Surgeons, 2004, pp 71-80.

100. A 23-year-old woman with a history of bilateral recurrent ankle sprains, progressive
cavovarus feet, and a family history of high arches and foot deformities is seen for
evaluation. Management consisting of bracing and physical therapy has been poorly
tolerated. Heel varus is partially corrected with a Coleman block. There are thick calluses
under the first metatarsal heads. Sensation to touch and Weinstein monofilament is normal.
Tibialis anterior and peroneus brevis are weak but present. What is the most appropriate
management?

1- Continued bracing, physical therapy, and Botox injections in the triceps surae
2- Peroneus longus to brevis transfer, medializing calcaneal osteotomy, and transfer of
the extensor digitorum longus to the peroneus tertius
3- Peroneus longus to brevis transfer, and transfer of the posterior tibial tendon to the
tibialis anterior tendon
4- Peroneus longus to brevis transfer, first metatarsal cuneiform dorsal closing wedge
osteotomy, and lateralizing calcaneal osteotomy with proximal translation
5- Triple arthrodesis
PREFERRED RESPONSE: 4

DISCUSSION: The history and presentation are consistent with type I Charcot-Marie-Tooth (CMT),
the most common form of hereditary peripheral motor sensory neuropathy. Type I CMT is the most
common, occurring in 50% of patients with CMT, and is characterized by marked slowing of motor
neuron velocities, and inconsistent slowing of sensory neuron velocities. Peroneus longus to brevis
transfer is indicated to release the overpull of the peroneus longus, and restore the eversion and
dorsiflexion function of the peroneus brevis. A lateralizing calcaneal osteotomy with proximal
translation is indicated to correct heel varus given that the Coleman block only allows for partial
correction of heel varus. Proximal translation of the posterior tuber corrects for the increased
calcaneal dorsiflexion, improving the lever arm for the triceps surae. A medial column closing
wedge osteotomy is often required to correct a rigid, or semirigid plantar flexed first ray to allow for
a balanced, plantigrade foot. Triple arthrodesis is indicated for rigid, arthritic hindfoot deformities.
Transfer of the posterior tibial tendon to the tibialis anterior is not indicated since it is an out-of-
phase transfer. Transfer of the posterior tibial tendon, when performed, should be to the lateral
aspect of the foot. A medializing calcaneal osteotomy would accentuate the heel varus. There is no
indication for Botox in CMT; Botox injection of the calf would further weaken push-off during gait.
Bracing of a progressive semirigid or rigid deformity is not recommended.

REFERENCES: Younger AS, Hansen ST Jr: Adult cavovarus foot. J Am Acad Orthop Surg
2005;13:302-315.
Sammarco GJ, Taylor R: Cavovarus foot treated with combined calcaneus and metatarsal
ostetotomies. Foot Ankle Int 2001;22:19-30.
Richardson EG (ed): Orthopaedic Knowledge Update: Foot and Ankle 3. Rosemont, IL,
American Academy of Orthopaedic Surgeons, 2003, pp 135-143.

Você também pode gostar